DE Module 2 - The Study of Calculus

You might also like

Download as pdf or txt
Download as pdf or txt
You are on page 1of 72

info@essu.edu.ph https://essu.edu.

ph
Borongan City, Eastern Samar, Phils., 6800

A Module in

THE STUDY OF CALCULUS


(A PRE–DIFFERENTIAL
EQUATIONS COURSE)
By Sir Ernesto Tan Anacta, BSCE, MSCE, Ph.D.CE

Module Contents:
 Part 1: Differential Calculus
 Part 2: Integral Calculus

This Module is a given to:


____________________________________

____________________________________

Student No.: __________________


Class Code No.: __________________
Contact No.: __________________
Address: _____________________
_____________________

By: Sir Ernesto T. Anacta


>>>May what you study be well studied.<<<
Certificate No. AJA19-0226

University Vision:
A technologically-advanced university producing professionals and competitive leaders
for local and national development.

University Mission:
To provide quality education responsive to the national and global needs focused on
generating knowledge and technology that will improve the lives of the people.

Core Values:
In pursuing its mission, goals and objectives the Eastern Samar State University officials,
faculty and students adhere to the following values:

 Excellence
Individual commitment to excellence is the central to the values that ESSU promotes.
The university will be able to achieve excellence through adherence to the highest
standards of performance and by collaborating with the very best in the fields of
instruction, research, extension and production.

 Accountability
Every member of the ESSU community as accountable for his every action, decision or
activities and for whatever money or property the university entrusts to him. He must
accept responsibility for whatever will be the consequences it may bring and to
disclose the results in a transparent manner. Thus, he must act with caution and
utmost consideration for ethics and honesty in the workplace.

 Service
Service is the commitment of the university to serve not only its stakeholders to
provide quality instruction, research, extension and production but also to serve the
need of every member of the ESSU community to advance their well-being.

Quality Policy:
We commit to provide quality instruction, research, extension and production grounded
on excellence, integrity and accountability as we move towards exceeding stakeholder’s
satisfaction in compliance with relevant requirements and well-defined continual
improvement measures.

“De kalidad nga edukasyon,


kinabuhi nga mainuswagon.”
A Module in

THE STUDY OF CALCULUS


(A PRE-DIFFERENTIAL EQUATIONS COURSE)
By Sir Ernesto Tan Anacta,

ABOUT THIS MODULE

Ihis module is the second of five parts for your Similar with Module 1 and in response to your
positive feedbacks and comments, your Sir
whole Differential Equations course. This is a
Anacta has likewise made this module
refresher of the two calculus courses which “conversational” in style. So – as usual – it will
you have taken up in your previous be all between you and me taking the “starring”
semesters: the Differential Calculus and role in this module. In this manner, you will feel
Integral Calculus. As what you will find out his presence every time you open the pages of
in our succeeding discussions, these two this module.
fields of calculus are the exact content of
Differential Equations. Hence, your Sir As usual, he has exerted all possible means
within his faculties to make this module flexible,
Anacta deemed it extremely necessary that
student-friendly, over-simplified and suited
we first revisit some of the topics that will for self-learning. The design, layout, concept
help us untangle some difficult topics which and contents of this module are all “Tatak Sir
you will encounter in your differential Anacta – Tatak ESSU” and no part of this
equations. module was copied in any manner from any
source.
As what your Sir Anacta has told you in your
Module 1, this second module is also aimed at The format, layout and style of Module 1 is
addressing the constraint which he always hereby replicated and followed in this module for
encounter in teaching the course – the you to feel continuity of their contents.
inadequate knowledge, background and Lastly, making this module in barely two-weeks-
preparation of some students on the pre- time (although challenging yet enjoyable) is very
requisite subjects which unwantedly result exacting and strenuous undertaking. Hence
unavoidable delays in presentation of topics your Sir Anacta solicits for your understanding
and, by domino effect, result to unfinished on some unintentional errors or omissions which
syllabus and backlog for the succeeding course. cannot be simply avoided.

HOW TO USE THIS MODULE

This module is a continuation of your first first step is Algebra and Trigonometry; second is
module and every part herewith compliment with Plane and Analytic Geometry; then Solid
the other parts in both modules. How you are Geometry; then Differential Calculus; then
going to use this module should be similar (or Integral Calculus; then Differential Equations
do it even better!) with the way you used your (which you are stepping now!); and finally,
first module, since there are no changes in Advanced Engineering Math – your last step.
format, style and presentation of contents. Your th
So you are now in your 7 step and all is left for
Sir Anacta’s piece of advice – though – is for you is a single step to finish your ascent (i.e.,
you make this module (and the rest of the other your series of mathematics subjects for your
four modules) as your “best friend” by opening BSCE course.) And, whether you like it or not,
the same more often. Remember that the higher you climb a ladder, the more nervous
mathematics is a ladder-type discipline. Your
3
you become because falling on the higher steps
is more hurting than falling on the lower steps – Module Content Color
much so that you are already on your second to Main Topic (F.x)
the last step! Besides, we are still in the “review” Sub-topic (F.x.x)
stage and the worse is still to come. Main Section (F.x.x.x)
Your Sir Anacta will not anymore elaborate on Sub-section (F.x.x.x.x)
the details of this section because it will just be Rule/Formula box
a repetition on Module 1. After all, we badly Figure/Table
need more space for the contents of this
Explanation
module. Just go to your Module 1 in case you
forgot some procedural details. However, for Discussion
your “on the spot” reference, the color coding Reminder Box
scheme is hereby reproduced to help you Remarks/Instructions
identify the parts of this module easily. Illustration
Examples
Self-Assessment Question
Self-Assessment Test
Pre-test/Post-Test
Bonus Problem

CONTENTS OF THIS MODULE

This module has 63 pages consisting of time as well as symbols, notations,


thirteen (13) parts as follows: abbreviations and acronym used in this module
and some answers to selected SATs.
A. About this module;
B. How to use this module; More specifically, the contents of this module
C. Module contents; with their respective pages are:
D. Introduction;
E. Overview; About this module. . . . . . . . . . . . . . . . 1
F. Instruction to students; How to use this module . . . . . . . . . . . . . 1
G. Objectives; Module contents . . . . . . . . . . . . . . . . 2
H. Pre-test; Introduction . . . . . . . . . . . . . . . . . . . 4
I. Discussion of contents; Overview . . . . . . . . . . . . . . . . . . . . 4
J. Post-test (exercises); Instruction to students . . . . . . . . . . . . . 5
K. Evaluation and Feedback; Objectives . . . . . . . . . . . . . . . . . . . . 6
L. Suggested readings; and Initial Evaluation (Pre-test) . . . . . . . . . . . 6
M. Attachments. Discussion of content . . . . . . . . . . . . . . 8
The Study of Calculus. . . . . . . . . . . . . . 8
Written on its pages are selected topics in both Types of Constants . . . . . . . . . . . . . . . 13
Differential Calculus and Integral Calculus which The Use of Differentiation Formulas. . . . . . . 17
are relevant to our study in Differential The Derivative Forms . . . . . . . . . . . . . . 17
Equations. To provide you with sufficient Direct Application . . . . . . . . . . . . . . . 17
preparations, further enrich your knowledge and Indirect Application . . . . . . . . . . . . . . 18
give you constant motivation, your Sir Anacta Power Formula . . . . . . . . . . . . . . . . 18
has included in its pages some 86 pre-test and Product Formula . . . . . . . . . . . . . . . 19
125 post-test problems, 20 sets of examples Quotient Formula . . . . . . . . . . . . . . . 20
containing 114 solved problems, 17 sets of The Differential Forms . . . . . . . . . . . . . 22
illustrations containing 125 items, six sets of Direct Application . . . . . . . . . . . . . . . 22
self-assessment tests (SAT’s) containing 42 Indirect Application . . . . . . . . . . . . . . 22
problems, 4 sets self-assessment questions The Two Methods of Differentiation . . . . . . . 23
(SAQs) containing 18 items, 4 sets of bonus Explicit Differentiation . . . . . . . . . . . . . 25
problems (BPs) worth P300 of cellphone Explicit differentiation on explicit equations . 26
Explicit Differentiation on implicit equations . 26
Also included in its Attachment are compilation Implicit Differentiation . . . . . . . . . . . . . 27
of 59 formulas for differentiation 64 formulas for The Two Methods of Integration . . . . . . . . 32
integration which you will be using from time to Integration Using Formulas . . . . . . . . . . 32

4
The Power Formula . . . . . . . . . . . . 32 Figure 2. . . . . 11 Figure 10 . . . . . 31
The Exponential Formula. . . . . . . . . . 33 Figure 3. . . . . 20 Figure 11 . . . . . 31
The Natural Logarithm Formula . . . . . . . 34 Figure 4. . . . . 27 Figure 12 . . . . . 31
Special Formulas . . . . . . . . . . . . . . 35 Figure 5. . . . . 29 Figure 13 . . . . . 41
The Integration Procedures . . . . . . . . . . 37 Figure 6. . . . . 30 Figure 14 . . . . . 44
Integration by Parts . . . . . . . . . . . . . 37 Figure 7. . . . . 30 Figure 15 . . . . . 45
Integration of Rational Fractions Figure 8. . . . . 30
by Partial Fractions . . . . . . . . . . . . 38
Trigonometric Substitution . . . . . . . . . 43 Tables:
Algebraic Substitution . . . . . . . . . . . 46 Table 1 . . . . . 16 Table 3 . . . . . . 34
Powers of Sines and Cosines . . . . . . . . 48 Table 2 . . . . . 31
Powers of Tangents and Secants . . . . . . 53 Equations:
Powers of Cotangents and Cosecants . . . 55 Eq. (1) . . . . . 11 Eq. (32) . . . . . 37
Illustrations: Eq. (2) . . . . . 11 Eq. (33) . . . . . 37
Illustration 1. . . 8 Illustration 9 . . . 16 Eq. (3) . . . . . 11 Eq. (34) . . . . . 38
Illustration 2. . . 9 Illustration 10 . . . 17 Eq. (4) . . . . . 11 Eq. (35) . . . . . 38
Illustration 3. . . 9 Illustration 11 . . . 17 Eq. (5) . . . . . 16 Eq. (36) . . . . . 38
Illustration 4. . . 11 Illustration 12 . . . 17 Eq. (6) . . . . . 17 Eq. (37) . . . . . 38
Illustration 5. . . 12 Illustration 13 . . . 18 Eq. (7) . . . . . 17 Eq. (38) . . . . . 38
Illustration 6. . . 12 Illustration 14 . . . 18 Eq. (8) . . . . . 18 Eq. (39) . . . . . 38
Illustration 7. . . 13 Illustration 15 . . . 18 Eq. (9) . . . . . 18 Eq. (40) . . . . . 38
Illustration 8. . . 16 Illustration 16 . . . 18 Eq. (10) . . . . 18 Eq. (41) . . . . . 38
Eq. (11) . . . . 18 Eq. (42) . . . . . 38
Examples: Eq. (12) . . . . 18 Eq. (43) . . . . . 38
Example 1 . . . 13 Example 11 . . . 28 Eq. (13) . . . . 19 Eq. (44) . . . . . 38
Example 2 . . . 14 Example 12 . . . 32 Eq. (14) . . . . 19 Eq. (45) . . . . . 38
Example 3 . . . 20 Example 13 . . . 35 Eq. (15) . . . . 20 Eq. (46) . . . . . 38
Example 4 . . . 21 Example 14 . . . 38 Eq. (16) . . . . 21 Eq. (47) . . . . . 38
Example 5 . . . 21 Example 15 . . . 41 Eq. (17) . . . . 21 Eq. (48) . . . . . 38
Example 6 . . . 22 Example 16 . . . 42 Eq. (18) . . . . 21 Eq. (49) . . . . . 38
Example 7 . . . 23 Example 17 . . . 43 Eq. (19) . . . . 21 Eq. (50) . . . . . 38
Example 8 . . . 24 Example 18 . . . 45 Eq. (20) . . . . 21 Eq. (51) . . . . . 38
Example 9 . . . 25 Example 19 . . . 45 Eq. (21) . . . . 22 Eq. (52) . . . . . 38
Example 10. . . 27 Example 20 . . . 46 Eq. (22) . . . . 31 Eq. (53) . . . . . 38
Eq. (23) . . . . 32 Eq. (54) . . . . . 42
Self-Assessment Questions (SAQs): Eq. (24) . . . . 32 Eq. (55) . . . . . 44
SAQ-1 . . . . . 10 SAQ-3 . . . . . . 17 Eq. (25) . . . . 32 Eq. (56) . . . . . 46
SAQ-2 . . . . . 14 SAQ-4 . . . . . . 25 Eq. (26) . . . . 32 Eq. (57) . . . . . 46
Self-Assessment Tests (SATs): Eq. (27) . . . . 32 Eq. (58) . . . . . 46
SAT-1 . . . . . 14 SAT-4 . . . . . . 28 Eq. (28) . . . . 32 Eq. (59) . . . . . 46
SAT-2 . . . . . 15 SAT-5 . . . . . . 35 Eq. (29) . . . . 32 Eq. (60) . . . . . 46
SAT-3 . . . . . 27 SAT-6 . . . . . . 36 Eq. (30) . . . . 37 Eq. (61) . . . . . 46
Eq. (31) . . . . 37
Bonus Problems (BPs):
BP-1 . . . . . . 15 BP-3 . . . . . . . 28 Final Evaluation (Post-test) . . . . . . . . . . . 47
BP-2 . . . . . . 27 BP-4 . . . . . . . 36 Evaluation and Feedback . . . . . . . . . . . . 50
Suggested Readings . . . . . . . . . . . . . . 51
Figures: Attachments . . . . . . . . . . . . . . . . . . . 52
Figure 1. . . . . 8 Figure 9 . . . . . 30

INTRODUCTION

Hello my dearly beloved students! Welcome to Modules 3, 4 and 5, “Everything in Differential


your second module: The Study of Calculus – Equations is about Calculus – both
A Review. As the title suggests, the content of Differential and Integral Calculus”.
this module will cover every topic in calculus
that you will be needing in your Differential The topics contained herein may not be totally
Equations course. As what you will find out in new to you (as you might have encountered this

5
already in your previous mathematics subjects). (although understandable) is a manifestation of
But based on actual experience of your Sir their unpreparedness, lack of knowledge and
Anacta with students of your level last year, he mathematical skill to face the course – for some
found out that most of them are not really reasons which may be beyond their control.
prepared to face the course per se because
they keep on asking some “hows” and “whys”, in Hence the need for this module.
the middle of discussion which, to him,

OVERVIEW

This module covers a brief yet comprehensive giving you a sort of motivation you will be given
review and discussions on selected topics on an initial evaluation in the form a multiple choice
both differential and integral calculus which will pre-test whose answers you must submit for
be the primary tool in solving differential feedback purposes.
equation problems. Here, you will be working on
methods of differentiating functions and In the middle of this module, you will also be
equations (including their higher and partial given SAQs, SATs and BPs to assess your level
derivatives) as well as integrating the same of understanding on the particular topic.
using both integration formulas, methods and
procedures. Finally, at the end of this module, you will be
Added to this module are comprehensive faced with a major exam (post-test) whose
illustrations, examples and solved problems, answers you must submit because the same will
with the inclusion of some relevant values and form part of your grade for this module. Just like
formulas used in coming up with the solution. in the case of Module 1, you will be also be
required to accomplish and submit Annex K
To test your preparedness in going into this (Evaluation and Feedback) which will be a
module by way of uncovering your pre-requisite requisite for the issuance of your Module 3.
skill, unlocking your expected difficulties and

INSTRUCTION TO STUDENTS

In using this module, please do the following: Sir Anacta anytime. His numbers are (Globe:
09063282771 and Smart: 09184021747).
1. Treat this module as your “best friend”
and “constant companion”. This module is 3. Be honest with yourself. Do not be a smart
your Sir Anacta’s personal gift to you. Although aleck. Pretending that you know a topic when
given to you for free, his time and effort spent in you don’t is not a good attitude. Note that this is
producing this module is deemed priceless as mathematics where there is no substitute for
everything in this module is a product of his own being an expert. After all, adding something to
mind and heart. Please see to it that you will your vocabulary will be your advantage in the
always take good care and preserve this module future.
until you finish your BSCE course. After all, you 4. Study all figures, illustrations and
will still be needing this during your review. examples intently and comprehensively
2. Thoroughly read and understand the including their ins and outs. They are meant
topics presented in every section. Do this in a emphasize, augment and expound the topic at
step-by-step manner because there are certain hand. Besides, they will provide you some
topics in the previous sections that complement insights and hints in answering questions and
with the succeeding sections. Do not escape nor solving tests, problems and exercises.
leave a section without clearing first yourself 5. Find your convenient self, time and place.
from doubts, nor finding answers to the Do not open the pages of this module if you are
annoying questions that will bother you in your not in an “eager-to-study” mood, if you are not
sleep. You can post your concerns, questions feeling well, nor when something is bothering
and queries on our online class “Sir Anacta – you. Neither do the same if you have more
Differential Equations CE Class”, which you important things to do. Do not make this module
have already registered, or feel free to text your as your “excuse” to doing your more relevant

6
undertakings. Remember that you are not time- first before solving the problems and solve first
pressured hence you are free to open its pages the problems before looking at their answers. If
at any time of the day, any day of the week – you are stuck in the middle of nowhere,
rain or shine. Find a place that is free from remember that your Sir Anacta is always there
disturbances and distractions, preferably an for your rescue. Just message him through text
open space with fresh air and good ambience or post in our online class so that he can help
and read this module there. The seashore, for you right away.
instance, is where your Sir Anacta cemented his
three degrees: BSCE, MSCE and Ph.D.CE. 8. Your Sir Anacta needs help. Finally, in his
desire to provide you an error-free module, it
6. Bear in mind that you are time-bounded, cannot be avoided that there may be some
meaning you need to finish this module within unintentional omissions, mistakes and
the allotted duration (only 14 days for this inaccuracies, either grammatically, numerically
module!). Remember that you have still four or computationally. For your information, the
modules to hurdle with no delay. So you must contents of this module are all his original idea
submit your requirements promptly on the given and was prepared by him in a very short period
deadline. Late submissions will be penalized.. of time spontaneously in front of a monitor – no
Incentive problems may be submitted ahead of books, no pencils and no papers. No content of
time but remember you have only one or two this whole module was copied in any manner
chance to submit. from any source. So please feel free to inform
him in case you found something correctible.
7. Do not place the cart ahead of the horse.
Study the concepts, illustrations and examples

OBJECTIVES

After completing this module you will be able 5. Derive the higher derivatives of a function by
to: successive differentiation;

1. Perform the two processes of calculus; 6. Evaluate the partial derivatives of functions;

2. Apply differentiation formulas in getting 6. Integrate functions using basic integration


derivatives of functions; formulas; and

3. Differentiate equations implicitly; 7. Evaluate integrands using integration


methods and procedures.
4. Differentiate functions explicitly;

INITIAL EVALUATION (PRE-TEST)

Instruction. This pre-test is worth P200 of cell


phone load (P100 each). Just select the letter of 3. The critical points are
the correct answer and text your answers to Sir a. (1, 11) c. (1, –16) e. (–2, 16)
Anacta using the prescribed Answers Format. b. (1, –11) d. (2, 16) f. NOTA

Test 1: 4. The inflection point is at

Given the function: y = 2x3 + 3x2 – 12x – 4 a. (½, 9) c. (–½,–9) e. (9, ½)


b. (–½, 9) d. (9, ½) f. NOTA
1. Its x-intercepts are
a. 2, 3.19, 0.31 f. –2, –3.19, 0.31 5. The slope at point (2, 0) is
a. 2 c. 6 e. 24
b. 2, –3.19, 0.31 e. 2, –3.19, –0.31
b. 4 d. 8 f. NOTA
c. 2, 3.19, –0.31 f. NOTA

2. The y-intercept is Test 2: Solve as required.


a. 0 c. 3 e. –4 A. Given the function p = (2q – 1)3
b. 2 d. –12 f. NOTA 6. Its first derivative p’ is
7
a. 2(2q – 1)2 c. 2(6q – 1)2 e. 6(2q – 1)2 b. 1 d. 2 f. NOTA
2 2
b. 4(2q – 1) d. 3(2q – 1) f. NOTA
Test 3: Find the derivative in differential form.
7. The value of p’ if q = 1 is
16. x2 – 3xy + y2 = 5
a. 0 c. 2 e. 6
a. (x – y) dx + (x – y) dy = 0
b. 1 d. 4 f. NOTA
b. (x – y) dx – (x – y) dy = 0
2v c. (x + y) dx + (x – y) dy = 0
B. Given: t =
2v d. (x – y) dx – (x + y) dy = 0
8. The first derivative t’ is e. (x + y) dx – (x + y) dy = 0
2 4 v f. NOTA
a. c. e.
(2  v) 2
( 2  v) 2 ( 2  v) 2
17. xey + e–x = 3
2 4 a. (ey + e–x) dx + xey dy = 0 f. NOTA
b. d. f. NOTA y –x y
(2  v) 2 (2  v) 2 b. (e + e ) dx – xe dy = 0
c. (ey – e–x) dx + xey dy = 0
9. The value of t’ if v = 1 is
d. (ey – e–x) dx – yex dy = 0
a. 2 c. 4 e. 1
e. (ey + e–x) dx + yexdy = 0
b. –2 d. –4 f. NOTA
18. w sin z + tan w = 
C. Given: x = (ey – e–y)2
a. (sin z + tan w) dw + w sin z dz = 0
10. The first derivative x’ is
b. (sin z + tan w) dw + w cos z dz = 0
a. ey – e–y c. 2(ey + e–y) e. e2y – e–2y
c. (sin z + sec2 w) dw + w sin z dz = 0
b. 2(ey – e–y) d. 2(e2y – e–2y) f. NOTA
d. (sin z + sec2 w) dw + w cos z dz = 0
11. The value of x’ if y = 0 is e. (sin z + sec2 w) dw + w tan z dz = 0
a. 0 c. –1 e. –2 f. NOTA
b. 1 d. 2 f. NOTA
19. ex sin y + ey + cos x = 1
D. Given: r = ln2 s + ln s2 a. (ex sin y – sin x) dx + (ex cos y + ey) dy = 0
b. (ex sin y + sin x) dx – (ex cos y + ey) dy = 0
12. The first derivative r’ is
c. (ex sin y – sin x) dx + (ex cos y – ey) dy = 0
a. 2 ln s + 2s c. ln s + 2s e. ln2 s + 1
d. (ex sin y + sin x) dx – (ex cos y + ey) dy = 0
2 1
b. (ln s + 1) d. (ln s + 1) f. NOTA e. (ex sin y – sin x) dx + (ex cos y – ey) dy = 0
s s
f. NOTA
13. The value of r’ if s = 1 is
20. r ln t + t ln r = 5
a. 0 c. –1 e. –2
t  r 
b. 1 d. 2 f. NOTA a.   ln r  dr +   ln t  dt = 0
r  t 
E. Given: L = e cos  r  r 
a.   ln r  dr +   ln t  dt = 0
14. Its first derivative L’ is  t   t 
a. e(cos  + sin ) f. e cos  t  t 
a.   ln r  dr +   ln t  dt = 0
 r   r 
b. e(cos  – sin ) e. e sin 
t  r 
c. –e(cos  + sin ) f. NOTA d.   ln t  dr +   ln r  dt = 0
r  t 
15. The value of r’ if  = 0 is r  t 
e.   ln t  dr +   ln r  dt = 0
a. 0 c. –1 e. –2 t  r 
8
f. NOTA a. 2 c. 4 e. 8
b. –2 d. –4 f. NOTA
Test 4: Find the indicated derivative.
21. y = x3 – 2x2 +3x + 4, find y’’’. 2z
30. Solve for in Item 1.
 x y
a. 2 c. 6 e. 24
b. 4 d. 8 f. NOTA a. 0 c. 4 e. 8
b. 2 d. 6 f. NOTA
22. x = e2y– 2e–y, find x’’.
a. 2e2y + 2e–y d. 2e2y + 4e–y Test 6: Evaluate the integrands.
b. 4e2y – 2e–y e. 2e2y – 4e–y
 1  2 x 
2 dx
c. 4e2y + 2e–y f. NOTA 31.

23.  = sin  + cos , find yIV.


a.
1
1  2 x 3 + c d. 
1
1  2 x 3 + c
a. sin  + cos  d. 2cos  – sin  2 3
b. sin  – cos  e. –cos  – sin 
b. 
1
1  2 x 3 + c e. 
1
1  2 x 3 + c
c. sin  + 2cos  f. NOTA 2 6

24. p = ye–y, find p’’’. c.


1
1  2 x 3 + c f. NOTA
3
a. ye–y + 3e–y c. –3ye–y –e–y
b. –ye–y + 3e–y d. –3ye–y + e–y
–y
c. ye – 3e –y
f. NOTA

32. x x 2  1dx

1 2 1 2
25. u = ev cos v find u”. a. ( x  1)1 / 2 + c d. ( x  1) 3 / 2 + c
2 3
a. ev sin v d. 2ev sin v
1 2 1 2
b. 2ev sin v e. –2ev sin v b. ( x  1) 3 / 2 + c e. ( x  1) 3 / 2 + c
2 6
c. 2ev cos v f. NOTA
1 2
c. ( x  1)1 / 2 + c f. NOTA
Test 5: Find the indicated partial derivative. 3

26. z = x2 – 2y2 + 3xy + 4x – 5y + 6, find


z
x
. 33.  
x x 2  1 dx 
a. 2x + 3y d. 2x – 3y 2 5/ 2 2 3/ 2 2 7 / 2 2 3/ 2
a. x  x +c d. x  x +c
b. 2x + 3y + 4 e. 2x – 3y – 4 5 3 7 3
c. 2x – 3y + 4 f. NOTA 2 5/ 2 2 3/ 2 2 7/ 2 2 5/ 2
b. x  x +c e. x  x +c
5 3 7 5
z
27. Solve for in Item 1. 2 2
y c. x 7 / 2  x 3 / 2 + c f. NOTA
7 3
a. 2x + 3y d. 2x – 3y


b. 2x + 3y + 4 e. 2x – 3y – 4 2 x dx
34.
c. 2x – 3y + 4 f. NOTA ( x 2  1) 2
1 2
a. +c d.  +c
2z x 1
2
x 1
2
28. Solve for in Item 1.
x 2 1 2x
b.  +c e. +c
a. 0 c. 4 e. 8 x 1
2
x 1
2

b. 2 d. 6 f. NOTA
2
c. +c f. NOTA
2
 z x 1
2
29. Solve for in Item 1.
y 2

9
e  x dx
e

x
35. ( 2  e x )3 dx
43.
e x  2
1 1
a. (2  e x ) 4 + c d.  (2  e x ) 4 + c
2 4 a. ln(e x  2) + c d.  2 ln(e x  2) + c

b. 
1
(2  e x ) 4 + c e. 
1
(2  e x ) 4 + c b.  ln(e x  2) + c e. ln(2e x  1) + c
2 6
c. 2 ln(e x  2) + c f. NOTA
1
c. (2  e x ) 4 + c f. NOTA
4
 x(ln x  1)
dx
44.

1 x  1
4 2
36. dx
a. x(ln x  1) + c d. ln(ln x  1) + c
a. 0 c. 4 e. 9
b. ln( x  1) + c e. ln x(ln x  1) + c
b. 1 d. 6 f. NOTA
c. x(ln x  x) + c f. NOTA
5


1 xdx
37. = ln( x 2  1)


2
0 x 1 2 csc2 x dx
45.
a. 0 c. 1.63 e. 2.63 cot x 1
b. 1.25 d. 2.25 f. NOTA a. ln(tan x  1) + c d. x(cot x  1) + c

38. 0 e
1 x 2
 1 dx  b. ln(csc x  1) + c e. x(csc x  1) + c
c. ln(cot x  1) + c f. NOTA
a. 0.50 c. 1.00 e. 1.75
46. xe( x 1)dx
2
b. 0.75 d. 1.5 f. NOTA


0 sin x  cos x 0 1  sin 2 x dx =3.14
2 dx = 1 2
1) 1 ( x 2 1)
39. a. e ( x +c d.  e +c
2
a. 0.14 c. 2.14 e. 4.14 2
1) 1 ( x 2 1)
b.  e ( x +c e. xe +c
b. 1.14 d. 3.14 f. NOTA 2
1 ( x 2 1)
 /2 c. e +c f. NOTA
40.  sin 2 x sin x dx 2
0

a. 0.34 c. 1.34 e. 3.34 47. (e x  e  x ) 2 dx



b. 0.67 d. 2.67 f. NOTA
a. 2(e x  e  x ) + c d. e 2 x  e 2 x  2x + c


2 dx
41.
1  2x b. e 2 x  e 2 x + c e. e 2 x  e 2 x  4x + c
a. ln(1  2 x) + c d. 2 ln(1  2 x) + c c. e 2x  e 2x  x + c f. NOTA
b.  ln(1  2 x) + c e. 4 ln(1  2 x) + c


e x 1
c. 2 ln(1  2 x) + c f. NOTA 48. dx
x


3 x 2 dx a. e x
 x+c d.
1
(e x  x ) + c
42.
x3  1 2
2
1 x
a. ln( x 2  1) + c d.  ln( x 3  1) + c b. e  x  + c e. 2(e x  x ) + c
2 
b.  ln( x 2  1) + c e. 3 ln( x 3  1) + c 2
c. 2e x
 +c f. NOTA
c. ln( x 3  1) + c f. NOTA x

10

(e x  1) 2
49.
e2 x
dx

55. cos 2 2 x dx

1 1 2 x 1 1
a. 2e  x  e 2 x + c d. x  2e  x  e +c a. ( x  cos 2 x) + c d. (2 x  sin 4 x) + c
2 2 2 4
1 1 2 x 1 1
b. 2e  x  e 2 x + c e. x  2e  x  e +c b. (2 x  sin 4 x) + c e. (2 x  cos 4 x) + c
2 2 2 4
1 1
c. 2e  x  e 2 x + c f. NOTA c. (2 x  cos 4 x) + c f. NOTA
2 2


ecos x
dx =  e cos x

50. 2 dx
cscx 56. dx
x 2 1
a. e cos x c. e cosx sin x e. e cosx csc x a. ln |x2 – 1| + c d. 2 arctan x+ c

b.  e cos x d. e cosx csc x f. NOTA 1 x


b. ln +c e. 2 arcsin x+ c
1 x
51.  sin(2 x  1) dx
x 1
1 1 c. ln +c f. NOTA
a. sin(2 x  1) + c d. cos(2 x  1) + c x 1
2 2


1 2 dx
b. 2 cos(2 x  1) + c e.  cos(2 x  1) + c 57. dx
2 x2  1
c. 2 cos(2x  1) + c f. NOTA a. ln |x2 – 1| + c d. 2 arctan x+ c


52. x cos(x 2  1) dx b. ln
1 x
1 x
+c e. 2 arcsin x+ c

1
a. 2 x sin(x 2  1) + c d. cos(x 2  1) + c x 1
2 c. ln +c f. NOTA
x 1
1 1
b. sin(x 2  1) + c e.  cos(x 2  1) + c


2 2 2 dx
58. dx
1
c.  sin(x 2  1) + c f. NOTA 1  x2
2 a. ln |x2 – 1| + c d. 2 arctan x+ c
53.  cot(1  2 x) dx 1 x
b. ln +c e. 2 arcsin x+ c
1 1 x
a. 2 tan (1  2 x) + c d. ln cos(1  2 x) + c
2
x 1
1 c. ln +c f. NOTA
b. 2 sec2 (1  2 x) + c e. ln sin(1  2 x) + c x 1
2


c. 2 csc2 (1  2 x) + c f. NOTA dx
59.
x2  x


2
54. sec (x  1) dx a. ln |x2 – x| + c d. (x2 – x)2 + c

1 1 x 2x 1
a. sin (x  1) + c d. tan(x  1) + c b. ln +c e. ln +c
  x 1 x2  x
1 1
b. cos (x  1) + c e. cot(x  1) + c x 1
  c. ln +c f. NOTA
x
1
c. csc2 (x  1) + c f. NOTA

11
60.  ln (2 x )dx __ / __ / __ / __ / __ (46 – 50)
__ / __ / __ / __ / __ (51 – 55)
a. x ln (2x) + c d. x (ln 2x – 1) + c __ / __ / __ / __ / __ (56 – 60)
b. ln (2x – 1) + c e. x (ln 2x + 1) + c
MECHANICS:
c. ln (2x + 1) + c f. NOTA 1. Two submissions are allowed.
(your 2nd submission is your final answer).
Answers Format: 2. Reward: P200 of cell phone load (P100 for
the first two perfect submissions.
0/00
__ / __ / __ / __ / __ (1 – 5) 3. In the absence of perfect submission, the
__ / __ / __ / __ / __ (6 – 10) P200 will be allotted as follows:
__ / __ / __ / __ / __ (11 – 15) st nd rd
P50/40/30 for 1 2 and 3 highest scores
__ / __ / __ / __ / __ (16 – 20) P20 for the four remaing highest scores
__ / __ / __ / __ / __ (21 – 25)
__ / __ / __ / __ / __ (26 – 30) 4.Deadline: September 16; Wed; 5:00 PM
__ / __ / __ / __ / __ (31 – 35) 5. Announcement of results will be made from
__ / __ / __ / __ / __ (36 – 40) time to time.
__ / __ / __ / __ / __ (41 – 45)

DISCUSSION OF CONTENTS

F.1 The Study of Calculus TheExplanation


left side box in Fig. 1 contains the function
or equation while the right side box contains its
Calculus is divided into five major branches derivative or differential. Think of these two
namely: Differential Calculus; Integral Calculus; boxes as two distinct places: the function box as
Differential Equations; Calculus of Variations; an origin and the derivative box as a destination.
and Calculus of Errors. (The last two branches The opposing middle arrows are the respective
are usually covered in specialized courses and routes towards these places which corresponds
are beyond the scope of our discussion. to the two processes of calculus: the Differential
Calculus (rightward arrow) and Integral Calculus
At this point in time, and before climbing to the (leftward arrow). Think of going to ESSU and
next steps towards our course, your Sir Anacta going back home. These are analogous to the
would like to remind you that the first two two processes of calculus.
branches of calculus mentioned above (the
Differential and the Integral Calculus) are the Remember This
exact content of Differential Equations. They are
(together with Algebra, Trigonometry, Analytic, In getting the derivative or differential of a
Plane and Solid Geometry) pre-requisites to function or equation, we use the process of
Differential Equations. Hence, your Sir Anacta differentiation; and in going back to the
will be relying on your mastery and expertise on function or equation, we use the process of
the above courses. integration or anti-differentiation.
Recall that these two branches of calculus are
reversible in process. This is shown in Figure 1. Illustration 1

Differentiation 1. The process of differentiation.


Function or Derivative or :
Equation Differential Consider the function
Integration
y = f(x) = x2 + 3x + 2 (1-a)
Figure 1 – The Process of Calculus Differentiating (1-a), we have
y’ = f’(x) = 2x + 3 (1-b)

12
or
dy
= 2x + 3 (1-c)  2 x  3dx +  4 y  1dy =0 (2-b)
dx
Eq. (1-c) can also be written in its differential 2x 2 4y2
– 3x + +y=c
form as 2 2
dy = (2x + 3) dx x2 + 2y2 – 3x + y = c (2-e)
or (2x + 3) dx – dy = 0 (1-d) Observe that Eqs. (1-a) and (1-f) as well as (2-
This means that from the function (1-a), we can e) and (2-f) actually differ by a constant. The
obtain its derivative (1-b) or (1-c) and, sooner has numerical (or fixed) constant “2”
subsequently, its differential (1-d) through the while the later contains an arbitrary constant “c”.
process of differentiation. In calculus, these two constants are treated
Similarly because their derivatives are both
Now, we will proceed with the backward route. zero. Thus, we can actually conclude that (1-a)
Starting with (1-d) and (1-f) are the same as far as the two
processes of calculus are concerned.
(2x + 3) dx – dy = 0 (1-d)
or dy = (2x + 3) dx (1-e) Types of Constants:

Integrating (1-e) and simplifying gives 1. Fixed or numerical constants. These are
constants with specific values (e.g., 0, 1, -3, ,
 dy =  2 x  3 dx e, tan 30, ln 3, 7 , etc.).
2
y = x + 3x + c (1-f) 2. Arbitrary constants. These are constants
which can take or be assigned with any value
where c = arbitrary constant.
except zero. In our course we will use the
This means that from the derivative we were symbols by a, b, c, ½a, eb, ln c, sin 2a, etc. for
able to go back to the original function using the arbitrary constants.
anti-differentiation (or integration) process.
x2 + 2y2 – 3x + y – 10 = 0 (2-a) F.1.1 Differentiation of
Differentiating (2-a), we have Functions and Equations
with Two Variables
2x dx + 4y dy – 3dx + dy = 0
(2x – 3) dx + (4y + 1) dy = 0 (2-b) In Section F.2.9 of our first module, we talked
about the two types of expressing functions and
from which we can obtain
equations with two variables (i.e., one DV and
dy one IV). These are
(4y + 1) + 2x – 3 = 0 (2-c)
dx y = f(x) (1)
(4y + 1)y’ + 2x – 3 = 0 (2-d) and f(x, y) = c (2)
This means that we were able to derive (2-b) The first one is the explicit form while the
and subsequently (2-c) and (2-d) by second one is the implicit form. In getting their
differentiation. derivatives, these two equations become

Very Important: You need to be “friendly” with y’ = f’(x) (3)


Eqs. (1-b), (1-c), (1-d), (2-b), (2-c) and (2-d)
because they will be your constant companions and f’(x, y) = 0 (4)
throughout the duration of this course. They are, where (’) is a derivative symbol. Eqs. (3) and (4)
in fact, differential equations! are the first derivative of (1) and (2). (Later on
we will discuss higher derivatives and partial
2. The process of integration. derivatives.) They are also called ordinary
: derivative or exact derivative because we are
Now, we will go back to where we belong. We
will start with (2-b), integrating (2-b) the same dealing with derivative of one dependent
and simplifying, thus variable ITO one independent variable. (Later in
this module, we will be dealing with partial

13
derivatives which is the derivative of functions a. The x-intercept is the point where the
with two independent vaiables. parabola crosses the x-axis. This can be found
by setting y = 0 in (1-b) and solving for the
Graphically, (3) and (4) represents the slope of values of x, thus
the tangent line at a certain point in a curve.
This is shown in Figure 2. 3 – 2x – x2 = 0  x2 + 2x – 3 = 0

Evaluating the derivatives of functions and (x – 1)(x + 3) = 0


equations with two variables may be x = 1, –3 Answer.
implemented by the derivative method or the
differential method using the derivative formulas Note: This means that the parabola crosses the
directly or indirectly. This will be our next x -axis at x = 1 and x = 3.
discussion.
b. The y-intercept is the point where the
y y’, dy/dx or f’(x, y) parabola crosses the y-axis which can be
located setting x = 0 in (1-a) and solving for the
value of y, this gives
P(x, y)
y y = f(x) or (0)2 + 2(0) + y = 3  y=3 Answer.
f(x, y) = c
Note: This means that the parabola crosses the
Curve y -axis at y = 3.

c. The critical point of a parabola is its vertex V.


0 x x
For this type of parabola, the tangent line at V is
a horizontal line and being a horizontal line, its
Figure 2 – Graphical Interpretation of slope is zero. Considering that the slope is the
Ordinary Derivative first derivative of the function, we have from (1-
b)
y’ = – 2 – 2x = 0 (1-c)
Example 1
which gives x = –1.
Given the equations:
And using this value in (1-b) gives
1. x2 + 2x + y = 3 3. y = x3 – x2 – 4x + 4
y = 3 – 2(– 1) – (– 1)2 = 4
2
2. y + 3x + 4y = 5
 The critical point (or vertex) is at (–1, 4).
Required: Answer.
a. Locate the x-intercept;
d. The inflection point is computed by setting the
nd
b. Locate the y-intercept; 2 derivative of (1-b) equal to zero and solving
c. Locate the critical point; for x, hence from (1-c) we have

d. Locate the inflection point; y” = –2 Answer.


e. Find the slope at x = 1; which is a constant.
f. Find the slope at y = 1; and Note: An inflection point is the location where
g. Show the graph. the curve changes its concavity (i.e., concave
upward ◡ or concave downward ◠, vice versa).
Solution: There is no such point in the case of a parabola
because, from the vertex, its graph stretches all
1. x2 + 2x + y = 3 (1-a) throughout in one direction only. Note that this is
Note: From your Module 1, you have learned confirmed by our answer –2 which is a constant
that this is an equation of a parabola with axis (not a point).
parallel to y-axis (meaning, it opens either up or e. For the slope at point x = 1, we have from (1-
down). Hence, this is a function which can be c)
transformed because its equation can be
transformed into y’ = – 2 – 2(1) = – 4 Answer.

y = 3 – 2x – x2 (1-b) Note: The slope at (1, 0) is negative (i. e.,


inclined to the left).

14
f. For the slope at y = 1, we need first to get the variable assignment” which means that we will
value of x corresponding to y = 1. Using (1-a) we treat x as the DV and y as the IV.
have
a. Setting y = 0 in (2-a), the x-intercept is
x2 + 2x + 1 = 3  x2 + 2x – 2 = 0 (1-d)
3x = 5  x = 5/3 = 1.67 Answer.
Solving (1-d) by the quadratic formula gives
Note: This means that the parabola crosses the
x = 0.732 and x = –2.732 x -axis at x = 1.67.
then using these values in (1-c), gives b. The y-intercept will be solved by setting x = 0
y’ = –3.46 and y’ = 3.46 Answer. in (2-a) and getting the value of y. We have
y2 + 4y – 5 = 0
Note: A pair of points in this parabola belonging
to the same horizontal line have the same (y – 1)(y + 5) = 0
magnitude in slope but opposite in direction.
y = 1, –5 Answer.
g. The graph using Desmos is shown below. Note: This means that the parabola crosses the
y -axis at y = 1 and y = –5.

c. The critical point (which is also its vertex V)


may be obtained by setting x’ = 0. Note that we
cannot express (2-a) as y = f(x) since the
equation is not a function. Also, the slope of the
tangent line is vertical which jives with x’ = 0. So
from (2-a), we have

1
x= (5  4 y  y 2 ) (2-b)
3
1
and x’ = (4  2 y ) = 0 (2-c)
3
which gives y = –2.

Summary: using these value in (2-b) gives

1. The equation is a function (parabola opening


down).
x=
1
3
 
5  4(2)  (2) 2 = 3

2. The critical point is its vertex, V(–1, 4).  The critical point is V(3, –2) Answer.
3. It crosses the x-axis (or x –intercepts) at x = 1 d. The inflection point will be computed by
and x = –3). setting x” = 0. So from (2-c) we have
4. It crosses the y–axis (y–intercept) at y = 3.
x’’ = –2/3 = –0.67 (2-d)
5. Its slope at x = 1 is –4 (inclined to the left).
which, like Item 1, is also a constant.
6. Its slope at y = 1 (corresponding to x = 0.73
and –1.73 are the same but oppositely e. For the slope at x = 1, we need first to get the
directed (–3.46 and 3.46, respectively. value of y corresponding to x = 1. Using (2-a) we
7. The curve has no inflection point. Meaning, have
the it has no reversal in its direction after the
y2 + 3(1) + 4y = 5  y2 + 4y – 2 = 0 (2-e)
vertex.
Solving (2-e) by quadratic formula gives
2. . y2 + 3x + 4y = 5 (2-a)
y = 0.45 and y = –4.45
Note: Recall in your Module 1 that this is a
then using (2-c), gives
parabola with axis parallel to x-axis (meaning, it
opens either left or right). Hence strictly x’ = –1.63 and x’ = 1.63 Answer.
speaking, this is not a function. But we can still
solve for the required values by “change of

15
Note: A pair of points in this parabola belonging
to the same vertical line have the same b. For the y-intercept, we have by setting x = 0 in
magnitude in slope but opposite in direction. (1-a):

f. For the slope at y = 1, we have:we have from y=4 Answer.


(2-c) Note: This means that the cubic crosses the y -
1 axis at y = 4.
x’ = (4  2 1) = –2 Answer.
3 c. The critical points of the curve are the
Note: The slope at (0, 1) is negative (i. e., maximum (or minimum) points. We will just set
inclined to the left). its slope (or first derivative) equation to zero
since the slope at this point is horizontal, that is:
g. The graph using Desmos is shown below.
y’ = 3x2 – 2x – 4 = 0 (3-b)
Solving for x in (3-b) using the quadratic formula
gives
x = –0.87 and x = 1.54
and using these values in (1-a) gives
y = 6.06 and y = –0.88
 The critical points are
(–0.87, 6.06)  max. point Answer.
(1.54, –0.88)  min. point Answer.
d. Solving for the inflection point, we have from
(3-b)
Summary: y” = 6x – 2 = 0
1. The equation is not a function (parabola or x = 1/3 = 0.33.
opening left).
Plugging in this value to (1-a) gives y = 2.59.
2. The critical point is its vertex, V(2, –3).
3. It crosses the x-axis (or x–intercept) at x =  The inflection point IP is at (0.33, 2.59).
1.67. Answer.

4. It crosses the y–axis (y–intercept) at y = 1 and Note: At IP the cubic changes its concavity;
y = –5. before IP, the curve is concave downward
(negative concavity); after IP, the curve is
6. Its slope at x = 1 (corresponding to y = 0.45
concave upward (positive concavity).
and –4.45 are the same but oppositely
directed (–1.63 and 1.63, respectively.
e. For the slope at point (1, 0), just substitute x =
5. Its slope at y = 1 is –2 (inclined to the left). 2 to (3-b), we have
7. Its inflection point is constant at –0.67. y’ = 3(1)2 – 2(1) – 4 = –3 Answer.
Meaning, the curve has no reversal in its
direction after the vertex. f. For the slope at point (0, –4), just substitute x
= 0 to (3-b):
3. y = x3 – x2 – 4x + 4 (3-a)
y’ = 3(0)2 – 2(0) – 4 = –4 Answer.
a. Recall that this is a cubic algebraic function.
Solving for its x-intercepts: f. For the slope at y = 1, we need first to get the
3 2
x – x – 4x + 4 = 0 values of x corresponding to y = 1. Using (3-a)
we have
(x + 1)(x + 2)( x – 2) = 0
1 = x3 – x2 – 4x + 4
x = –1, –2, 2 Answer.
or x3 – x2 – 4x + 3 = 0 (3-d)
Note: This means that the curve crosses the the
x -axis three times: at x = –1, –2 and 2.
16
Solving (3-d) by NR method (or just use e. Find the slope at x = 1;
Desmos), gives
f. Find the slope at y = 1; and
x = –1.91, 0.71 and 2.20
g. Show the graph.
then using these values in (3-b), gives
y’ = –5.91, –3.91 and 6.12. Answer. F.1.1.1 The Use of
Differentiation Formulas
g. The graph using Desmos is shown below.
For your easy and quick reference, your Sir
Anacta has provided you a roster of
differentiation formulas of functions in both their
derivative and differential forms, and both for
direct and indirect applications in Attachment F.
For the sake of refreshing your memory and
helping you in tackling Part 1 of our course, your
Sir Anacta will give you some illustrations on
how to use these formulas.

F.1.1.1.1 Derivative Forms

The differentiation formulas in their derivative


forms (Column 1) include (D-1) to D-30) for
direct application and (D-31) to (D-56) for
Summary: indirect application. These formulas uses the
fractional form of derivative d / dx . Note that we
1. The equation is a function (a cubic). can actually transform the said form into its
2. It has two critical points: (–0.87, 6.06) which is prime and the differential operator notations
the maximum point and (1.54, –0.88) which (which we will also talk about in our succeeding
is the minimum point. topics). We will discuss these two methods
3. It crosses the x-axis (or x –intercepts) at three here.
locations: x = –2, x = 1 and x = 2).
Discussion
4. It crosses the y–axis (y–intercept) once at y =
4. 1. Direct Application. In the direct application,
5. Its slope at x = 1 is –3 (inclined to the left). a single variable, say x, is involved in the
formula. Here, you can replace x with another
6. Its slope at y = 1 corresponding to x = –1.91,
letter or symbol since the same is only a dummy
0.71 and 2.20 is –5.91, –3.91 and 6.12,
variable. In applying these formulas, the
respectively.
variables on the LHS of the formula are being
7. The curve has one inflection point at (0.33, carried directly into its RHS. Consider the
2.59) which is the boundary of its concavity. following examples.
To the left of this point, the curve is concave
downward and upward to its right. Example 2
Self-Assessment Test 1 Find the first derivative.
3
Given the equations: 1. y = x2 – 3x + 4 3. w = z z
1. x2 + y2 – 5y = 14 3. x3 – 8x – 2y + 3 = 0 1 2
2. r = (n -1)2(n2 + 1) 4. t = v + +
3 2
2. x = y – 4y + 8 v v2
Required: 5. = sec  + cot 

a. Locate the x-intercept(s); Solution:


b. Locate the y-intercept(s); 2
1. y = x – 3x + 4 (1-a)
c. Locate the critical point(s);
Using (D-4), (D-3) and (D-1) in (1-a) gives
d. Locate the inflection point;

17
dy that “x” is merely a dummy variable which can
= 2x – 3 Answer. be replaced with other variables.
dx
2 2
Since this is just a review, your Sir Anacta will
2. r = (n -1) (n + 1) (2-a) just give you an illustration on how to use the
Simplifying (2-1), we have three very important formulas: the Power
Formula (D-31), Product Formula (D-37) and
r = (n2 - 2n + 1)(n2 + 1) Quotient Formula (D-38) using. The other
formulas follow suit.
= n4 – 2n3 + n2 + n2 – 2n + 1
r = n4 – 2n3 + 2n2 – 2n + 1 D-31: The Power Formula
and using (D-4), (D-3) and (D-1) gives d n du
(u ) = n un–1 n  0, 1; u = f(x)
dr dx dx
= 4n3 – 6n2 + 4n – 2 Answer.
dn
The power formula is used to get the
derivative of a function raised to an exponent
3
3. w = z z (3-a) n, where n can be any number (plus or
minus), except 0 and 1. Observe that the
Simplifying (3-a): exponent n is prefixed in the answer and
subtracted by 1 in its exponent. The function
3 3 32
w= z z1 2 = z is further differentiated WRT the independent
variable x.
w= z  3 / 2 1/ 3 = z1/ 2 = z Example 3
and using (D-9) gives
Obtain the derivatives.
dw 1 1. y = (2x – 3)3 4. s = ln4 (3t + 1)
= Answer.
dz 2 z
2. r = 1  3s 2 5.  = (sin 2 – 1)2
1 2 1
4. t = v + + = v + + 2v–2 (4-a) 3.  = (e –2 + 1)2
v v2 v
Solution:
Using (D-2), (D-8) and (D-4) and simplifying, we
have
1. y = (2x – 3)3 (1-a)
dt 1 1 du
=1+ + 2(–2)v–2–1 = 1 – – 4v–3 Let: u = 2x – 3, n = 3, =2
dv 2 2 dx
v v
dt 1 4 Substituting these values to (D-31) and
=1– – Answer. simplifying, we have
2
dv v v3
dy
5.  = sec  + cot  (5-a) = 3(2x – 3)3–1(2) = 6(2x – 3)2 Answer.
dx
Using (D-17) and (D-16) in (5-a) gives Do This: Expand (1-a), use (D-1), (D-3) and (D-
d 4) and compare the two answers.
= tan  sec  – csc2  Answer.
d
2. r = 1  3s 2 = (1 – 3s2)1/2
It’s time to move to the second form.
du
Let: u = 1 – 3s2, n = ½, = –6s
2. Indirect Application. If the direct application ds
cannot be used anymore due to complexity of
variables in a function, a method using Substituting these values to (D-31) and
substitution or transfer of variable, say u and/or simplifying, we have
v to get the a derivative is usually resorted to. As dr
shown in formulas (D-31) through (D-56), the = ½ (1 – 3s2) 1/2–1(–6s) = – 3s (1 – 3s2)–1/2
functions u = f(x) and v = f(x). Be reminded again ds

18
nd nd
dr 3s times the derivative of the 2 factor plus the 2
= Answer. st
factor times the derivative of the 1 factor.”
ds 1  3s 2
–2
Example 4
3.  = (e + 1)
2

Solve the following using (D-37).


du
Let: u = e–2 + 1, n = 2, = –2e–2
d 1. y = (2 – x)(x – 3)2 4. p = q2esin 2q
Substituting these values to (D-31) and 2. w = m3e–2m 5. x = z3 ln (2z + 1)
simplifying:
3. t = e–2s tan 2s
d
= 3(e–2 + 1)2–1(–2e–2) = –6e–2(e–2 + 1) Solution:
d
2
d 6 1. y = (2 – x)(x – 3)
=– (e2 + 1) Answer.
d 4 Let: u = (2 – x) v = (x – 3)2
e
4
4. s = ln (3t + 1) = [ln (3t + 1)]
4 du dv
= (2 – x) = 2(x – 3)
dx dx
du 3
Let: u = ln (3t + 1), n = 4, =
dt 3t  1 Substituting these values to (D-37) and
simplifying, we have
Substituting these values to (D-31) and
simplifying, we have dy
= (2 – x)2(x – 3) + (x – 3)2(2 – x)
dx
ds 3
= 4[ln (3t + 1)]4–1 = (2 – x)(x – 3)(2 + x – 3)
dt 3t  1
dy
12 = (2 – x)(x – 3)(x – 1) Answer.
= [ln (3t + 1)]3 dx
3t  1
3 –2m
2. w = m e
ds 8 ln 3 (3t  1)
= Answer.
dt 3t  1 Let: u = m3 v = e–2m

5.  = (sin 2 – 1)2 du dv
(5-a) = 3m2 = –e–2m
dm dm
du
Let: u = sin 2 – 1, n = 1, = 2 cos 2 Substituting these values to (D-37) and
d
simplifying, we have
Substituting these values to (D-31) and
dw
simplifying, we have = (m3)(–e–2m) + (e–2m)(3m2)
dm
d
= 2(sin 2 – 1)2–1(2 cos 2) = m2e–2m(–m + 3)
d
dw
d = m2(3 – m)e–2m Answer.
= 4 cos 2(sin 2 – 1) Answer. dm
d
Do This: Expand (5-a), use (D-1), (D-3) and (D- 3. t = e–2s tan 2s
4) and compare your results. Let: u = e–2s v = tan 2s
D-37: The Product Formula du dv
= –e–2s = 2 sec2 2s
d dv du ds ds
(uv) = u +v ; u = f(x), v = f(x).
dx dx dx Substituting these values to (D-37) and
simplifying, we have
The product formula is used to get the
derivative of two (or more) functions. This is dt
st = (e–2s)(2 sec2 2s) + (tan 2s)(–e–2s)
easily remembered in words as: “The 1 factor ds

19
dt Solution:
= e–2s(2 sec2 2s – tan 2s) Answer.
ds ( x  1) 2
1. y = (1-a)
2 sin 2q x 1
4. p = q e
Let: u = q2 v = esin 2q Let: u = (x + 1)2 v=x–1
du dv du dv
= 2q = 2 esin 2q cos 2q : = 2(x + 1) =1
dq dq dx dx
Substituting these values to (D-37) and Substituting these values to (D-38) and
simplifying, we have simplifying, we have

dp dy 2( x  1)( x  1)  ( x  1) 2 (1)
= q2(2 esin 2q cos 2q) + (esin 2q)(2q) =
dq dx ( x  1) 2
= 2qesin 2q(q + 1) ( x  1)[ 2( x  1)  ( x  1)]
=
dp ( x  1) 2
= 2q(q + 1)esin 2q Answer.
dq
( x  1)( 2 x  2  x  1)
=
3
5. x = z ln (2z + 1) ( x  1) 2

Let: u = z3 v = ln (2z + 1) dy ( x  1)( x  3)


= Answer 1.
du dv 2 dx ( x  1) 2
= 3z2 =
dz dz 2 z  1 Alternative Solution: Using logarithm.
Substituting these values to (D-37) and Taking the “ln” of (1-a):
simplifying, we have
( x  1) 2
dx 2z 3 ln y = ln (1-b)
= + 3z2 ln (2z + 1) Answer. x 1
dz 2 z  1
but ln (A/B) = ln A – ln B (1-c)

D-38: The Quotient Formula then applying (1-c) into (1-a):

du dv ln y = ln (x + 1)2 – ln (x – 1)
v u
d u dx dx ; u = f(x), v = f(x). or ln y = 2 ln (x + 1) – ln (x – 1) (1-d)
  =
dx  v  v 2
Differentiating (1-d) and simplifying:
The quotient formula is used to get the 1 dy 2 1
derivative of the quotient of two functions. = –
y dx x  1 x  1
This is easily remembered in words as: “The
denominator times the derivative of the 2( x  1)  ( x  1) 2x  2  x  1
numerator minus the numerator times the = =
( x  1)( x  1) ( x  1)( x  1)
derivative of the denominator over the
denominator squared.” ( x  3)
dy
= y (1-e)
dx ( x  1)( x  1)
Example 5
Plugging in (1-a) into (1-e) and simplifying
Differentiate using (D-38).
dy ( x  3) ( x  1) 2
=
( x  1) 2
et  1 m3 dx ( x  1)( x  1) x  1
1. y = 3.  = 4. w =
x 1 e 2t e 2m
dy ( x  1)( x  3)
z 1 sin   1 = Answer 2.
2. y = 5.  = dx ( x  1) 2
z2 1 cos   1
Observation: Answers 1 and 2 are the same!

20
z 1 Question: Are Answers 1 and 2 the same?
2. y =
z2 1 m3
4. w =
e 2m
Let: u = z + 1 v = z2 1
Note: This is similar with Item 2 of Example 3.
du dv z
= 1 = Let: u = m3 v = e2m
dz dz 2
z 1 du dv
= 3m2 = e2m
Note that we used (D-31) in getting dv/dz. dm dm
Substituting these values to (D-38) and
simplifying: Substituting these values to (D-38) and
simplifying, we have
z 2  11  z  1
z
dw (e 2m )(3m 2 )  (m 3 )( 2e 2m )
dy z 1
2 =
= dm (e 2 m ) 2
dz 2
 z2 1
 
  m 2 e 2 m (3  2 m )
=
dy z2 1 z2  z (e 2 m ) 2

 
=
dz z2 1 z2 1 dw m 2 (3  2m)
= Answer.
dm e 2m
dy 1 z
 
= Answer.
dz z2 1 z2 1 sin   1
5.  =
cos   1

et  1 Let: u = sin  – 1 v = cos  – 1


3.  = (3-a)
e 2t du
= cos 
dv
= –sin 
d d
Let: u = et + 1 v= e2t
Substituting these values to (D-38) and
du dv simplifying, we have
= et = 2e2t
dt dt d (cos   1)(cos  )  (sin   1)(  sin  )
=
Substituting these values to (D-38) and d (cos   1) 2
simplifying, we have
d cos2   cos   sin 2   sin 
d (e 2t )(e t )  (e t  1)( 2e 2t ) =
= d (cos   1) 2
dt ( e 2t ) 2
d 1  cos   sin 
e 2 t ( e t  2e t  2)  et  2 = Answer.
= = d (cos   1) 2
( e 2t ) 2 e 2t
Before we proceed with the next topic, please
d et  2 try solving the following problems.
=  Answer 1.
dt e 2t
Self-Assessment Test 2
Alternative Solution: Reducing first (3-a)
Find the derivatives.
et  1 et 1
 = = + = et – 2t + e–2t
e 2t e 2t e 2t 1. r = s 2 s 2  1 4. y = (x2 + 3)2(x – 3)3

 = e–t + e–2t (3-b)


2.  =
cos   sin 
5.  =
e 2n
1  2

finally differentiating (3-b) gives cos 2 e 3n


d 3. x = ln (2z2 + 1)2
= –e–t – 2e–2t Answer 2.
dt

21
dr = (4n3 – 6n2 + 4n – 2) dn Answer.
Bonus Problem 1
3
For P50-worth of load. (@ P25 each), find y’. 3. w = z z (3-a)
Just text your answers to your Sir Anacta. Only
one trial per student which will be the final Eq (3-a) reduces to:
answer.
w = z (3-b)

1. y = x2 3 x x 2. z = w2 ln2 (2w2 + 1)2 and using (D-9) in (3-b) gives


1
dw = dz Answer.
F.1.1.1.2 Differential Forms 2 z
Ihe differentiation formulas in their differential
1 2 1
forms are listed in Column 2 of Attachment F. 4. t = v + + = v + + 2v–2 (4-a)
These include (D-1) to D-30) for the direct v v2 v
application and (D-31) to (D-56) for the indirect
Using (D-2), (D-8) and (D-4) in (4-a) and
application. Note that these formulas are similar
simplifying, we have
to the derivative forms except for their format
wherein “d” (which stands for differential) is 1 1
dt = [1 + + 2(–2)v–2–1] = 1 – – 4v–3
used instead of “d/dx”. In terms of application, v 2
v 2
these formulas are used similarly with the
derivative forms. We will discuss these two  1 4 
dt = 1    dv Answer.
applications here.  v 2
v3 
Discussion 5.  = sec  cot  + cos  sec  (5-a)

1. Direct Application. Similar with the direct Reducing (5-a) we have


application in derivative forms, a single variable
1 cos 1
(x in the formulas) is also used in their  = + cos 
differential forms. You can also use any variable cos sin  cos
in lieu of x. In applying the formula, the variables
1
on the LHS of the formula are copied exactly  = + 1 = csc  + 1 (5-b)
into the RHS of the equation. sin 
Note: For our succeeding examples, we will just and using (D-18) and (D-1) gives
use the problems in Examples 1 and 2, for us to
compare the results. We will also exclude the d = –cot  csc  d Answer.
reduction and simplification of these equations.
Instead, we will go directly to the derivation of 2. Indirect Application. Due to the limitation of
their answers. the direct application formulas, derivatives of
equations containing complex variables are
Example 6 solved using the indirect application listed in the
nd
2 column of (D-31) to (D-56) wherein
Using Example 1 problems, find the derivatives substitution or transfer of variable, say u and/or
in differential form. v is resorted to in order to get the derivative. In
these formulas, u = f(x) and v = f(x) and x may be
Solution: replaced with other variables.

2
1. y = x – 3x + 4 (1-a) Example 7
Using (D-4), (D-3) and (D-1) in (1-a) gives Obtain the derivatives in differential form of the
dy = (2x – 3) dx Answer. equations in Example 2.

Solution:
2. r = (n -1)2(n2 + 1) (2-a)
Eq. (2-a) reduces to 1. y = (2x – 3)3 (1-a)

r = n4 – 2n3 + 2n2 – 2n + 1 du
Let: u = 2x – 3, n = 3, =2
dx
Using (D-4), (D-3) and (D-1) gives

22
Substituting these values to (D-31) and F.1.1.2 The Two Methods of
simplifying, we have Differentiation
dy = 6(2x – 3)2 dx Answer.
There are two methods by which equations
may be differentiated depending on its type.
2. r = 1  3s 2 = (1 – 3s2)1/2 These are explicit differentiation and implicit
du differentiation. We will discuss them here in
Let: u = 1 – 3s2, n = ½, = –6s details, but bear in mind that this is just a
ds refresher and you can go to the next topic if you
Substituting these values to (D-31) and are already familiar with this.
simplifying, we have
F.1.1.2.1 Explicit Differentiation
dr
= ½ (1 – 3s2) 1/2–1(–6s) = – 3s (1 – 3s2)–1/2
ds Explicit Differentiation is used to get the
derivative of equations (functions included) in
3s both explicit and implicit forms which are,
dr = ds Answer.
1  3s 2 respectively, written in the form of
y = f(x) (1)
3.  = (e–2 + 1)2
and f(x, y) = c (2)
du
Let: u = e–2 + 1, n = 2, = –2e–2 In the explicit form [Eq. (1)], the dependent
d variable (DV) y is explicitly expressed in terms of
the independent variable (IV) x. Here, DV is
Substituting these values to (D-31) and
always differentiated with respect to the IV. In its
simplifying:
derivative form, (1) is written as
d = –6e–2( e–2 + 1) d Answer.
dy
4 4
y’ = = f’(x) (5)
4. s = ln (3t + 1) = [ln (2t + 1)] dx
du 2 Functional notation
Let: u = ln (2t + 1), n = 4,=
dt 2t  1 Fractional notation
Substituting these values to (D-31) and Prime notation
simplifying, we have
In the case of implicit form [Eq. (2)], the two
4–1 2 variables are combined on the left hand side of
ds = 4[ln (2t + 1)] ] dt the equation. Hence we cannot directly pinpoint
2t  1
which is dependent variable or independent
 8 ln 3 ( 2t  1)  variable. All we can do is just assume one as
ds =   dt Answer. dependent variable and the other one as
 2 t  1 
  independent variable. In carrying out their
differentiation, we can use both the prime and
5.  = (sin 2 – 1)2 (5-a) the fractional notations shown in Eq. (5).

Let:u = sin 2 – 1, n = 1,
du
= 2 cos 2 Discussion
d
1. Explicit differentiation on explicit t
Substituting these values to (D-31) and equations. Consider an equation in its explicit
simplifying, we have form:
d = 4 cos 2(sin 2 – 1) d Answer. y = f(x) (1)

Self-Assessment Test 3 Rule 1: Differentiate each side of Eq. (1)


with respect to “x”, applying the appropriate
Using the same problems in Self-Assessment differentiation formulas in their derivative
Test 1, find their derivatives of the equations in forms (Attachment F; Column 1).
differential form using the differential form
formulas. The derivative of (1) is

23
dy  1
y’ = = f’(x) (6) 2. –  = 0   –1 =  ( + 2)
dx  2

Important:  =  2 + 2 + 1

1. The IV “x” and DV “y” in Eq. (1) are merely d


= 2 + 2 = 2( + 1) Answer.
dummy variables, which means that they can d
be replaced by other symbols.
2 2
2. The three derivative notations in Eq. (3) may 3. x (y – 1) = y(x – 1) (3-a)
be used alternatively depending on the Reducing (3-a) into its explicit form we have
problem at hand. If the independent variable
is worth mentioning or relevant to the x2y – x2 = y(x2 – 2x + 1)
problem and if the same needs to be x2y – x2 = x2y – 2xy + y
nd rd
evaluated, the 2 and 3 notations in (3)
must be used. If, on the other hand, the IV is x2
st –x2 = – y(2x – 1)  y= (3-b)
not relevant in the problem, the 1 notation of 2x  1
(3) may suffice. This notation, however, does
not mean that there is no IV in the equation. Applying (D-38) in (3-b) and simplifying, we
The same is only “implied” which means that have
“any” letter or symbol may be used as IV.
(The relevance of this will crop up when we dy 2 x2 x  1  2 x 2
=
go to Part 2 of our course.) dx 2x  12
3. As a general procedure, convert first the
equation into (1). If the same cannot 4x 2  2x  2x 2 2x 2  2x
= =
converted, use Rules 2a and 2b. 2 x  12 2 x  12
4. Your Sir Anacta will rely on your mastery of
dy 2 x( x  1)
getting the derivative of functions and = Answer.
equations, particularly in using the formulas dx 2 x  12
listed in the Attachment.
2x x
4. e – ye + x = 0 (4-a)
Example 8
x  e 2x
Find the derivatives. yex = x + e2x  y= (4-b)
ex
 1
1. x2 – xy – 2x + y + 1 = 0 2. +=0 We will solve (4-b) by two methods.
 2
2 2
3. x (y – 1) = y(x – 1) 4. e2x – yex + x = 0
Method 1: Applying (D-38) in (4-b) and
5.  sin  – cos2  +  = 0 simplifying, we have

Solution: (e x )(1  2e 2 x )  ( x  e 2 x )(e x )


y’ =
Note: Rule 1 dictates that we must first
e  x 2

transform the given equation into its explicit form e x  2e 3 x  xe x  e 3 x


or Eq. (1). y’ =
e2x
1. x2 – xy + 2x – y + 1 = 0 (2-a)
e 3 x  xe x  e x e x (e 2 x  x  1)
y’ = =
Transforming (2-a) into its explicit form e2x e2x
x2 + 2x + 1 – xy – y = 0 e2x  x  1
y’ = Answer 1.
(x + 1)2 – y(x + 1) = 0 ex
x+1–y=0  y=x+1 (2-b)
Method 2: Reducing (4-b) further
Using (D-1) and (D-2) in (2-b) gives
x  e 2x x e2x
y’ = 1 Answer. y= = +
ex ex ex

24
y = xe–x + ex (4-c) Example 9
and applying (D-37) and (D-6) in (4-c) gives Obtain the derivatives of the following equations
y’ = x (–e–x) + e–x(1) + ex using Rules 1 and 2 and compare the results
with Example 1.
y’ = –xe–x + e–x + ex Answer 2.
x2  1
1. y = x3 – 2x2 + 3x + 4 3. y =
Do This: Prove that Answers 1 and 2 are the x2 1
same. 2. x2 – xy + 2x – y + 1 = 0 4. e2x – yex + x = 0
5.  sin  – cos2  +  = 0 (5-a) 5.  sin  – cos2  +  = 0

Reducing (5-a) into its explicit form and applying Solution:


(D-1) and (D-13) we have
Note: We will solve the above problems in a
cos 2  1  sin 2  “direct-to-the-point” manner since this is just a
 = =
sin   1 sin   1 repetition of Example 1. What your Sir Anacta is

 =
1  sin  1  sin   trying to teach you is how to apply Rules 2a and
2b in coming up with the correct answers. Here
1  sin  we go…
 = 1 – sin  3 2
1. y = x – 2x + 3x + 4 (1-a)
d
= – cos  Answer. 3 2
or x – 2x + 3x + 4 – y = 0 (1-b)
d
Rule 2a: Assuming “y” as the DV in (1-b), we
We will now proceed with the next topic. have
dx dx dx dy
2. Explicit differentiation on implicit _ 3x2 – 4x +3 +4– =0
equations. Consider an equation in the form of dx dx dx dy
dy
f(x, y) = c (2) 3x2(1) – 4x(1) + 3(1) + 4 – =0
dy
Unlike in the case of Eq. (1) wherein the one
dx
variable is explicitly expressed in terms of the (3x2 – 4x + 3) – =0 (1-c)
other, the two types of variables in Eq. (2) are dy
implied. Meaning, we cannot directly tell which dy
of the two variables are dependent variable or = 3x2 – 4x + 3 Answer 1.
independent variable. But still, we can get its dx
derivative by assuming any one of them as the
Rule 2b: Assuming “x” as the DV in (1-b), we
dependent variable (the other one, of course,
have
will be the independent variable) and
differentiate Eq. (2) with respect to the chosen x3 – 2x2 + 3x + 4 – y = 0
independent variable. To be specific, we will
discuss the two possible options: dx dx dx dy
3x2 – 4x +3 – =0
dy dy dy dy
Rule 2a: Assume “y” as the DV in (2).
dx
(3x2 – 4x + 3) –1=0
Differentiate each term WRT x, putting: dy
dx/dx = 1 to every “x” differentiated and dx 1
= Answer 2.
dy/dx or (y’) to every “y” differentiated. dy 3x  4 x  3
2

Note: Comparing the answer of Item 1


Rule 2b: Assume “x” as the DV in (2). (Example 3) with the two answers above, we
have
Differentiate each term WRT y, putting:
1. y’ = 3x2 – 4x + 3 Item 1; Example 3
dy/dy = 1 to every “y” differentiated and
dy
dx/dy or (x’) to every “x” differentiated. 2. = 3x2 – 4x + 3 Answer 1.
dx

25
dx 1 other. So they are the same since dy dx =
3. = Answer 2.
dy 3x 2  4 x  3 1
.
dx dy
The three answers above are the same since
dy 1 x2  1
y’ = = = 3x2 – 4x + 3. 3. y = (3-a)
dx dx dy x2 1
Decomposing (3-a), we have
2. x2 – xy + 2x – y + 1 = 0 (2-a)
y(x2 – 1) = x2 – 1
Rule 2a: Assuming “y” as the DV in (2-a), we
have x 2y – x 2 – y + 1 = 0 (3-b)
2x(1) – [xy’ + y(1)] + 2(1) – y’ = 0 Rule 2a: Assuming “y” as the DV in (3-b), we
have
2x – xy’ – y + 2 – y’ = 0
x 2y – x 2 – y + 1 = 0
xy’ + y’= 2x – y + 2
[x2y’ + 2xy(1)] – 2x(1) – y’ = 0
(x + 1)y’ = 2x – y + 2
x2y’ + 2xy – 2x – y’ = 0
2x  y  2
y’ = Answer 1. (x2 – 1)y’ + 2xy – 2x = 0
x 1
Rule 2b: Assuming “x” as the DV in (2-a), we (x2 – 1)y’ = 2x – 2xy = 2x(1 – y)
have
2 x(1  y )
x2 – xy + 2x – y + 1 = 0 y’ = Answer.
x2 1
2xx’ – [x(1)+ yx’] + 2x’ – 1 = 0
Rule 2b: Assuming “x” as the DV in (3-b), we
2xx’ – x – yx’ + 2x’ – 1 = 0 have
(2x – y + 2)x’ = x + 1 x 2y – x 2 – y + 1 = 0

x 1 [x2(1) + y(2xx’)] – 2x(x’) – 1 = 0


x’ = Answer 2.
2x  y  2 x2 + 2xyx’ – 2xx’ – 1 = 0

Note: Let us verify if the answer of Item 2 2xx’ – 2xyx’ = x2 – 1


(Example 3) and the two answers above are the 2x (1 – y)x’ = x2 – 1
same. We have
1. y’ = 1 x2 1
Item 2 (Example 3) x’ = Answer.
2 x(1  y )
2x  y  2
2. y’ = Answer 1.
x 1 Question: Can you verify if the answers above
and that of Item 3 (Example 3) are the same?
x 1
3. x’ = Answer 2.
2x  y  2 4. e2x – yex + x = 0 (4-a)

From Item 2 (Example 3), we derived Rule 2a: Assuming “y” as the DV in (4-a), we
have
y=x+1 (2-b)
2e2x (1) – [yex(1) + ex(y’)] + 1 = 0
Substituting (2-b) to Item 2 above, we have
2e2x – yex – ex(y’) + 1 = 0 (4-b)
2 x  ( x  1)  2 2 x  x  1  2
y’ = = Multiplying (4-b) by e–x and simplifying
x 1 x 1
2ex – y – y’ + e–x = 0
x 1
y’ = =1 Answer.
x 1 y’ = 2ex + e–x – y
which is the answer of Item 2 (Example 3). Rule 2b: Assuming “x” as the DV in (4-a), we
Items 2 and 3 above are reciprocal with each have
2e2x (x’) – [yex (x’) + ex (1] + x’ = 0

26
2e2x (x’) – yex (x’) – ex + x’ = 0
L 1
2. L2 = 3. e2z – 2zet + 1 = 0
T 1
(2e2x – yex + 1)x’ – ex = 0
4.  sin  – cos  +  = 0
x
e
x’ = Answer.  m  1
2e 2x
 ye x  1 5. ln   + 2m = 1
 n 1 
Do This: Prove that the answers Item 4 B. Find the indicated derivative using
(Example 3) and this item are the same. Rule 2.
By this time, your Sir Anacta is presuming that
you are already very familiar with the two rules. 1. (x2 – y)2 + (x – y2)2 = 1, Find y’.
For this reason, he will just present his
succeeding examples in a “lesser detailed” ex  y
2. y = , Find x’.
manner without mentioning the steps anymore ey  x
and using another variables.
3. ers + res + ser + e2 = 0, Find s’.
5.  sin  – cos  +  = 0
2
(5-a)
4. 2 cos  –  sin2  +  = 2, Find ’
Differentiating WRT  in (5-a):
5. m ln n – n ln m + ln (mn) = 2 ln 2, Find n’.
[ cos  (1) + (sin )’]
F.1.1.2.2 Implicit Differentiation
– [2 cos  (–sin )(1) + ’ = 0
Implicit Differentiation is used to get the
 cos  + ’ sin  + 2 sin  cos  + ’ = 0
differential of equations in their implicit form [Eq.
(sin  + 1) ’ +  cos  + 2 sin  cos  = 0 (2)]. Here, both the DV and IV are differentiated
separately. The answer will be in the form of
 cos  sin 2
’ =  Answer 1. M(x, y) dx + N(x,y) dy = 0 (7)
sin   1
where M(x, y) and N(x,y) are coefficients of of
Differentiating WRT  in (5-a): differential of dx and dy, respectively. Your Sir
 sin  – cos2  +  = 0 Anacta recommends that you familiarize and
remember this equation because this is exactly
[ cos  (’) + (sin )(1)] the type of equation which we will be dealing
with in Part 2 of our course. Here goes the
– [2 cos  (–sin )(’) + 1 = 0
rule…
 cos  (’) + sin 
Rule 3: Differentiate each term in (2) WRT
+ (2 sin  cos )(’) + 1 = 0 to both variables “x” and “y” and applying
( cos  + 2 sin  cos )(’) + sin  + 1 = 0 differentiation formulas in their differential
form (Attachment F, Column 2), placing to
( cos + sin 2 )(’) + sin  + 1 = 0 each term:

sin   1 “dx” to every “x” differentiated and


’ =  Answer 2.
 cos  sin 2 “dy” to every “y” differentiated.

Do This: Verify if the answers of Item 7


Example 10
(Example 3) and Answer 1 above are the same
that is Obtain the derivative of the following equations
 cos  sin 2 using Rule 3.
– cos    .
sin   1 1. x2 – xy + 2x – y + 1 = 0
2. e2x – yex + x = 0
Self-Assessment Test 4
3.  sin  – cos2  +  = 0
A. Find the derivatives using Rule 1.
2
4. ln(uv) – ln u2 + v = 1
1. y – 2xy + 3x + y + 1 = 0
5. r2 + 2s2 – 4r + 3s + 5 = 0

27
Solution: 4. x3y2 + x2y3 – 2x2y2 + xy2 = 10

1. x2 – xy + 2x – y + 1 = 0 5.  sin  – cos2  +  = 0

2x dx – (x dy + y dx) + 2 dx – dy = 0 Bonus Problem 2


2x dx – x dy – y dx + 2 dx – dy = 0
For P30-cell phone load (each problem) find the
(2x – y + 2) dx – (x + 1) dy = 0 Answer. differential form (4) of the following equations
using Rule 3.
Question: Is this answer similar with the answer
of Item 2 (Example 2) which is 1. (x3 + y2)2 = 4xy2

2x  y  2 2. 2 sin 2 – e cos2  + e = 0


dy
2. y’ = = ?
dx x 1 3. y ln (x/y) – x ln (y/x) + x/y = e

e2x – yex + x = 0 Let’s move on to our next topic…


2x x x
2e dx – [ye dx + e dy] + dx = 0
F.1.1.3 Higher Derivatives
2e2x dx – yex dx – ex dy + dx = 0
So far, what we have covered was purely first-
(2e2x – yex + 1) dx – ex dy = 0 Answer. order derivatives and first-order differentiation.
In this section, we will be dealing with higher
Question: Can you covert this answer to the derivatives which can only be performed
answer of Item 5 of Example 2? through successive differentiation. Meaning
3.  sin  – cos2  +  = 0 from the given function or equation, we will first
st nd rd
get its 1 derivative, then its 2 , then its 3 , its
th
( cos  d –  sin  d) 4 , and so on. We will designate the letter “n”
as the highest number the function is
– (–2 cos  sin  d) + d = 0 differentiated. This is shown in Table 1.
 cos  d –  sin  d + sin 2 d + d = 0
Table 1. Details of Higher Derivatives
( cos  + sin 2) d – ( sin  – 1)d = 0
Derivatives
Answer.
n Fractional Prime
2 Details
4. ln(uv) – ln u + v = 1 Notation Notation
ln u + ln v – 2 ln u + v = 1 1
d
y dy
y’
dx dx
ln u + ln v – 2 ln u + v = 1 d  dy  d2y
2   y”
du dv du dx  dx  dx 2
+ – 2 + dv = 0
u v u d 2  dy  d3y
3   y’’’
v du + u dv – 2v du + uv dv = 0 dx 2  dx  dx3
–v du + u dv + uv dv = 0 d 3  dy  d4y
4   yIV
v du – u(1 + v) dv = 0 Answer. dx3  dx  dx 4

d 4  dy  d5y
2 2
5. r + 2s – 4r + 3s + 5 = 0 5   yV
dx 4  dx  dx 5

2r dr + 4s ds – 4 dr + 3 ds = 0 … … … …
n 1
(2r – 4) dr + (4s + 3) ds = 0 Answer. d  dy  n
d y
n
n 1
  y(n)
dx  dx  dx n

Self-Assessment Test 5
Notes:
Find the differential form (4) of the following
equations using Rule 3. 1. The fractional notation was introduced by
Gottfried Wilhelm Liebniz (1646–1716) a and
1. (r – 2s)2 + (rs – 1)2 = 0 2. r tan  +  er =  the prime notation was first used by Isaac
3. (x2 – y)2 + (x – y2)2 = 1 Newton (1642–1727).

28
2. In some literatures, dot notation is used in lieu
z’ = 2e2t – 3e–3t z’’’ = 8e2t – 27e–3t
of prime notation, i.e., y, y , y , y , and so on.
z” = 4e2t + 9e–3t z(IV) = 16e2t + 81e–3t
3. In the case of prime and dot notation, a
rd
roman numeral is used after the 3 derivative. Answer.
4. r = sin  + cos (2 + )
Example 11
dr
=  cos  – 2 sin (2 + )
Find the indicated derivative. d
1. y = x5 – 2x4 + 3x3 – 4x2 + 5x + 6, find y(VI).
d 2r
= –2 sin  – 4 cos (2 + )
d2y d 2
2. y = 1  x , find
2
.
dx 2
3. z = e2t + e–3t, find z(IV).
d 3r
= –3 cos  + 8 sin (2 + ) Answer.
d r 3 d 3
4. r = sin  + cos (2 + ), find .
d 3 5. s = sec  + tan 
d 2s
5. s = sec  + tan , find . ds
d2 = sec  tan  + sec2 
d
Solution:
d 2s
5 4 3 2 = sec  (sec2 ) + tan  (sec  tan )
1. y = x – 2x + 3x – 4x + 5x + 6 d 2

y’ = 5x4 – 8x3 – 9x2 – 8x + 5 + 2 sec  (sec  tan )


y” = 20x3 – 24x2 – 18x – 8 d 2s
= sec  (sec2  + tan2  + 2 sec  tan )
y’’’ = 60x – 48x – 18 
2
y (IV)
= 120x – 48 d 2

y(V) = 120  y(VI) = 0 Answer. d 2s


= sec  (sec  + tan )2 Answer.
d 2
2. y = 1  x 2 = (1  x 2 )1 / 2
Application of Higher Derivatives:
dy 1
= (1  x 2 ) 1 / 2 (–2x)
dx 2 In physical applications, the function is used
to represent displacement s, the first
rd
dy
=  x(1  x 2 ) 1 / 2
derivative as velocity v and the 3 derivative
dx as acceleration a. That is
y  displacement, s
d2y   1
=   x  (1  x 2 ) 3 / 2 (2 x)
dy
  2
2
dx y’ = y =  velocity, v
 (1  x )2 3 / 2
(1)  dx


=  x 2 (1  x 2 ) 3 / 2  (1  x 2 ) 1 / 2  y” = y =
d2y
dx 2
 acceleration, a


=  (1  x 2 ) 3 / 2 x 2  (1  x 2 ) 
Example 12
=  (1  x 2 ) 3 / 2 (1)
1. A particle moves according to the equation s
2 = t3 + 4t – 3 meters. Find its displacement,
d y 1 1
= = velocity and acceleration after 2 seconds.
2 2 3/ 2
dx (1  x ) (1  x 2 ) 3
2. The path of an object in a plane is governed
Answer. by r =  sin  + 2 cos  radians. Find its
displacement, velocity and acceleration after 1
3. z = e2t + e–3t second.
29
Solution: Graphically, (10) and (11) represents the slope
of the two tangent lines at a certain point in a
1. s = t3 – 4t + 3 = (2)3 – 4(2) + 3 = 3 m z
surface as shown in Figure 3. Note that is
x
2 2
v = 3t – 4 = 3(2) – 4 = 8 m/s
a = 6t = 6(2) = 12 m/s2 Answers. z
the tangent line // to the y-axis while is the
y
2. s =  sin  + 2 cos 
tangent line // to the y-axis.
s = (1) sin 1 + 2 cos (1) = 1.92 rad
v =  cos  + sin  – 2 sin 
z z = f(x, y) or
v =  cos  – sin  z
f(x, y, z) = 0
z y
v = (1) cos (1) – sin (1) = –0.30 rad/s Answers.
z
a = – sin  + cos  – cos  Surface x
a = – sin  P(x, y, z)
a = –(1) sin (1) = –0.84 rad/s 2
Answers. 0
y y
Question: How do we get the derivative of x
functions and equations containing three x
variables?
Figure 3 – Graphical Interpretation
Answer: This will be our next topic.
of Partial Derivative

F.1.2 Differentiation of F.1.2.1 Partial Differentiation


Functions and Equations
with Three Variables As previously mentioned, partial differentiation
(PD) is only applicable to functions with two IVs.
In Section F.2.9 of our first module, we also
We will state the rules here.
covered the two types of functions and
equations containing three variables (i.e., one Partial Differentiation Rules:
DV and two IVs). These are
z = f(x, y) (8) Given the function in the form of
z = f(x, y) (8)
and f(x, y, z) = c (9)
Rule 1: To obtain the first partial derivative
The first one is the explicit form while the
second one is the implicit form. Differentiation of of z WRT x, differentiate (8) WRT x using
these type of functions is totally different from exact differentiation, treating y as constant.
functions with two variables. Here, we resort to z
partial differentiation. Its form is = f(x’, y) (PD-1)
x
z Rule 2: To obtain the first partial derivative
= f(x’, y) (10)
x of z WRT y, differentiate (8) WRT y using
exact differentiation, treating x as constant.
read as “partial derivative of z WRT x.” z
= f(x, y’) (PD-2)
y
z
and = f(x, y’) (11)
y Rule 3: To obtain the second partial
derivative of of z WRT x, get the partial
read as “partial derivative of z WRT y.” where () derivative of (PD-1), following similar
is a partial derivative symbol. Eqs. (10) and (11) procedure with Rule 1.
are called “partial derivatives” because we are
dealing here with two IVs (x and y). The process 2z   z 
of getting the derivative of a function with two =   = f(x”, y) (PD-3)
x 2 x  x 
IVs is called partial differentiation.

30
Using Rule 3 from (1-b):
Rule 4: To obtain the second partial
derivative of of z WRT y, get the partial 2z   z  
=  = (4x3 + 6xy2 – 4y + 5)
derivative of (PD-2), following similar x 2 x  x  x
procedure with Rule 2.
2z
2z   z  = 12x2 + 6(1)y2 = 6(2x2 + y2) (1-d)
=   = f(x, y”) (PD-4) x 2
y 2 y  y 
Using Rule 4 from (1-c):
Rule 5: To get the partial derivative of (PD-
2z   z  
1) WRT y, get the partial derivative of (PD-1), =   = (–6y2 + 6x2y – 4x + 6)
y 2 x  x  y
following similar procedure with Rule 2.
2z   z  2z
=   = f(x’, y’) (PD-5) = –12y + 6x2(1) – 0 + 0
y x  y   x  y 2

Rule 6: To obtain the partial derivative of 2z


= –12y + 6x2 = 6(x2 – 2y) (1-e)
(PD-2) WRT x, get the partial derivative of y 2
(PD-2), following similar procedure with Rule
1. Using Rule 5 from (1-b):
2z   z    z  2z   z  
=  =   = f(x’, y’) (PD-6)  = (4x3 + 6xy2 – 4y + 5)
xy x  y  y  x 
=
 y  x  y   x  y

2z
Note: The derivative of “constant variable” is = 0 + 6x(2y) – 4 = 12xy – 4 (1-f)
zero. y x

Using Rule 6 from (1-c):


Example 13
2z   z  
Find the six partial derivatives of the following: =   = (–6y2 + 6x2y – 4x + 6)
xy x  y  x
1. z = x4 – 2y3 + 3x2y2 – 4xy + 5x + 6y + 7
2z
2. z = e2xy + e3xe2y + e(2x – 3y) + e3x – e2y – 5 = 0 + 6(2x)y – 4 = 12xy – 4 (1-g)
xy
3. z = sin (xy) – x cos y + y sin x
Observe: (1-f) and (1-g) are the same.
+ sin x cos y + sin x – cos y + 1
Note: Your Sir Anacta will present the solution
Solution: of the succeeding problems in a “less-detailed”
Applying Rules 1 to 6 in succession: manner. He urges you, therefore, to familiarize
first six rules above and Example 1 before
4 3 2 2
1. z = x – 2y + 3x y – 4xy + 5x + 6y + 7 (1-a) proceeding.
Using Rule 1 in (1-a): 2. z = e2xy + e3xe2y + e(2x – 3y) + e3x – e2y – 5
z
= 4x3 – 0 + 3(2x)y2 – 4(1)y + 5(1) + 0 + 0 z
x = e2xy(2y) + (3e3x)e2y + e(2x – 3y)(2)
x
z + 3e3x + 0 + 0
= 4x3 + 6xy2 – 4y + 5 (1-b)
x z
= 2ye2xy + 3e3xe2y + 2e(2x – 3y) + 3e3x Answer.
Using Rule 2 in (1-a): x
z z
= –2(3y2) + 3x2(2y) – 4x(1) + 0 + 6(1) + 0 = e2xy(2x) + e3x(2e2y) + e(2x – 3y)(–3)
y y
+ 0 – e2y(2) – 0
z
= –6y2 + 6x2y – 4x + 6 (1-c) z
y = 2xe2xy + 2e3xe2y – 3e(2x – 3y) – 2e2y Answer.
y

31
2z F.1.3 The Two Methods
= 2y[e2xy(2y)] + 3[e3x (3)]e2y
x 2 of Integration
+ 2e(2x – 3y)(2) + 3e3x(3) Our previous topic was all about obtaining the
 z
2 derivative or differential of equations (including
= 4y2e2xy + 9e3xe2y + 4e(2x – 3y) + 9e3x Answer. functions). This means that, given an equation,
x 2
we arrived at their derivative or differential via
2z differentiation. To complete the cycle shown in
= 4xe2xy(2x) + 2e3xe2y(2) Figure 1, we will now go the other way around.
y 2 Here, starting with their derivatives or
– 3e(2x – 3y)(–3) – 2e2y(2) differentials, we will return back to the function
or equation via integration or anti-differentiation.
2z
= 4x2e2xy + 4e3xe2y + 9e(2x – 3y) – 4e2y Answer. This is actually the complete picture of our
y 2 course – the Differential Equations.

2z For this particular section, we will just touch the


= 2[y(e2xy)(2x) + ye2xy(1)] topics that will lessen your burden in studying
y x Part 3 of this course, considering the very wide
+ 3e3x (e2y)(2) + 2e(2x – 3y)(–3) + 0 scope of Integral Calculus.
2z
= 4xye2xy + 2ye2xy + 6e3xe2y – 6e(2x – 3y) F.1.3.1 Integration Using
y x
Formulas
Answer.
Included in Attachment G are 57 integration
 z
2 formulas which will help you in solving
= 4[x(e2xy)(2y) + e2xy(1)] + 2e3x(3)e2y integration problems. Aware of the fact that
xy
memorizing them is next to impossible, he will
– 3e(2x – 3y)(2) – 0 just illustrate to you how to apply these
2z formulas. Note that similar to the differentiation
= 4xye2xy + 4e2xy + 6e3xe2y – 6e(2x – 3y) formulas, the integration formulas also use “u”
xy
as the variable which can be a function of other
Answer. variables.
 z
2
 z
2
Observe: = Important:
yx xy
1. Use only the formula that is applicable to the
problem at hand.
3. z = sin (xy) – x cos y + y sin x
2. The differential of your assigned u and du
+ sin x cos y + sin x – cos y + 1
should differ by a constant only.
Note: Your Sir Anacta will just give you the 3. The reciprocal of the constant pertained to in
answers for you to derive. No. 2 is called neutralizing factor nf which
should always be included in your solution.
z
= y cos (xy) – cos y + y sin x
x Note: Our succeeding examples will just
+ cos x cos y + cos x include the application of the most common
and important formulas. Each solution will be
2z presented in a straightforward and less-
= –y2 sin (xy) + y cos x – sin x cos y – sin x
x 2 detailed manner to save space. Just text your
Sir Anacta (or post on our virtual class) your
z questions or additional queries.
= x cos (xy) + x sin y + sin x – sin x sin y + sin y
y
I-4: Power Formula
 z
2
= –x2 sin (xy) + x cos y – sin x cos y + cos y u n 1
y
 u du = nf
2 n
+ c n  0, –1; u = f(x)
n 1
2z 2z
= = –xy sin (xy) + cos (xy) + sin y The power formula is used to integrate a
yx xy
function raised to an exponent n, where n can
+ sin x – cos x sin y
be any number (plus or minus), except 0, –1.

32
Observe that the answer is in fractional form
with its exponent n added by 1 as the
+  sin x dx
denominator. The neutralizing factor nf is Let u = cos x, du = –sin x, nf = –1
always affixed to the answer.
1 2
=  cos3 x – cos2 x – cos x + c
Example 14 3 2
1
Integrate using the Power Formula. =  cos3 x – cos2 x – cos x + c Answer.
3


(cos x  1)2 dx
 (2 x  1) 2 / 3 dx

1. 4.
5. x ln 3 (1  x 2 ) Let u = ln (1 - x2)
csc x dx
x2  1 2 xdx
du =

 x 1 dx
3 2
x ln (1  x ) 1 x2
 x(2 x ln 1  x  
2
 1)2 / 3 dx

2. 5. 2 3
xdx
2 = 2 xdx
x 1 2 =
x 2 1


x
ln 1  x 
e dx
3. dx 2 4 nf = 1/2
3 x
e 1 = +c
4
1 4
Solution: = ln (1 – x2) + c Answer.
4
Let u = 2x – 1, du = 2 dx
 (2x 1)
1. 2/3
dx
nf = 1/2 Self-Assessment Test 7

1 ( 2 x  1) 31 1 Integrate using the power formula.


= + c = (2 x  1) 4 + c Answer.
2 3 1 8

x 2dx
1.
 x(2  3x 2 )3 / 2 dx 4.
9 x3 1
2.
 x(2 x 2  1) 2 / 3 dx Let u = 2x2 – 1, du = 4x dx
nf = 1/4


2


  dx
2. e x  e x  1 dx 5.
 (2x 1)  x dx
2 2/3
=   x ln 3 (3x  3)

=
1 (2 x 2  1)2 / 31
2 2 / 3 1
+c Answer. 
3. sin 2 x sin 2 x dx

I-6: Exponential Formula


Let u = ex – 1

e x dx e x dx
3. dx =  dx du = ex dx
e
u
3 x x
 1/ 3 du = nf  eu + c, u = f(x)
e 1 ( e 1) nf = 1

 e 1  e dx =  e  1
The exponential formula is used to integrate an
1/ 3 1 / 3
= x x x
ex dx exponential function with base e (e  2.71828).
Observe that its answer, affixed with a
neutralizing factor, is the same with the
(e x  1) 1 / 31 (e x  1) 2 / 3 integrand.
= (1) +c= +c
1 / 3  1 2/3
Example 15
3 x
= (e – 1)2/3 + c Answer.
2 Integrate using the Exponential Formula.

2e(sin x 1)
4.

(cos x  1) 2 dx
csc x
1.
 e (13x) dx 4.
 sec x
dx

e(ln 2 x  1)
  xe (3 x 1) dx

2
= (cos 2 x  2 cos x  1) sin x dx dx 2. 5. dx
2x


= cos2 x  sin x dx + 2  cos x  sin x dx
3.

e 2 x 3
dx
2x  3

33
Solution: If the integrand can be transformed into
fractional notation where its numerator is an
exact differential of the denominator, the
e
(13 x) 1 (1 – 3x)
1. dx = 
e +c Answer. natural logarithm formula comes into rescue.
3
Be careful, though, that this formula is very
Let u = 1 – 3x, du = –3 dx, nf = –1/3
specific: if the denominator is u then the
numerator “MUST” be du, no more no less.
 xe e
(3 x 2 1) (3 x 2  1)
2. dx =  x dx
Example 16
Let u = 3x2 – 1, du = 6x dx, nf = 1/6
Integrate using the Natural Logarithm Formula.
1 (3 x 2  1)

sec2 2 xdx

= e +c Answer. dx
6 1. 4.
3x 1 1  tan 2 x
dx


2 x  3 , du =
 x
e 2 x 3 x 2 dx 2
ln x(ln x  1)dx

Let u =
3. dx 2x  3 2. 5.
2x  3 nf = 1 1  2x 3


(2e 2 x  1)dx
e
2 x 3 dx 3.
=  = 2x  3 + c x  e2 x
2x  3
Answer. Solution:

2e(sin x 1)
  3x 1 = 3 ln (3x + 1) + c
dx 1
4. dx = 2  e (sin x1)  cos x dx 1. Answer.
sec x

Let u = sin x – 1, du = cos x dx, nf = 1 Let u = 3x + 1, du = 3 dx, nf = 1/3

= 2e(sin x – 1) + c

Answer. 2. x 2 dx 1
=  ln (1 – 2x3) + c
1  2x 3 6
e(ln 2 x  1)

1 (ln 2 x  1) dx
5.
2x
dx =
2 e 
x
Answer.
Let u = 1 – 2x3, du = –6x2 dx, nf = –1/6
dx
Let u = ln 2x + 1, du = , nf = 1


x (2e 2 x  1)dx
3. = –ln (x – e2x) + c Answer.
1 x  e2 x
= e(ln 2x + 1) + c Answer.
2 Let u = x – e2x, du = (1 – 2e2x) dx
= –(2e2x – 1) dx, nf = –1
Self-Assessment Test 6
sec2 2 xdx

Integrate using the exponential formula. 4. 1
= ln (1 + tan 2x) + c Answer.
1  tan 2 x 2
e  2 xe
(1 2 x) ( 2 x 3)
2
1. dx 3. dx
Let u = 1 + tan 2x, du = 2 sec2 2x dx, nf = 1/2

e 3x 1 dx
2.
 3x  1
dx 5.  (1  cos x) e
xsin x
dx 5.

ln x(ln 2 x  1)dx
x
Let u = ln2 x + 1
du =
2 ln x
dx
x
2e(sec 2 x 1) tan 2 x
 e (1 ln x ) dx

2

 (ln
2 ln xdx nf = 1/2
4. 6. dx = x  1) 
cos 2 x x
1
= ln (ln2 x + 1) + c Answer.
I-12: The Natural Logarithm Formula 2
du
 u
= nf  ln u + c, u = f(x) Self-Assessment Test 7
Integrate using the natural logarithm formula.

34
sec2 ln x 
 
xdx


3x 2  2
 2 x3
3dx 5. 12. dx
1. 4. dx x2  6 x 2x
32 x3

6.
 x2  2 x
dx
13.  arcsinx
2 
 1 dx

x
e 2 x dx
e
x  cos 2 x
2. dx 5. dx


2

2x  1  sin 2 x  1
 5  2x 2 e
dx x
7. 14. arctan 1  2e x dx
3.  cot 2 x dx

Solution:
And now for the special formulas…


Special Formulas:
1. 3  x 2 dx Let u2 = x2 u=x du = dx
2
u nf = 1 a =3 a= 3
I-22:  a 2  u 2 du =
2
a2  u2
Using I-22:
2
a u x 3 x
+
2
arcsin + c
a
= 3  x 2 + arcsin +c Answer.
2 2 3
du 1 u
I-24:
 a2  u2 = arctan +c Let u2 = 4x2 u = 2x

du = 2dx
a a 2. 1  4 x 2 dx
a2 = 1 a=1 nf = 1/2
du 1 ua
I-25:
 u2  a2 =
2a
ln
ua
+c Using I-22:

1  2x 1 2x 
du 1 au =  1  4 x 2 + arcsin 
I-26:  a 2  u 2 = 2 a ln a  u + c 2  2 2 1 

1 1  1
 x 1  4 x  arcsin 2 x  + arcsin 2x) + c
2
I-35:  secu du = ln |sin u + tan u| + c =
2 2  2
Answer.
1 1
 cos
2

I-38: u du = u + sin 2u + c dx
2 4 3. Let u2 = 5x2 nf = 1/ 5
2
9  5x
u= 5x a2 = 9
 sec
I-41: 2
u du = tan u + c
du = 5 dx a=3
I-55:  arcsin u du = u arcsin u + 1 u2 + c Using I-24:
1 5x
= arctan +c Answer.
 arctanu du = u arctan u – ½ ln|u + 1|
2
I-57: 3 3

Let u2 = 4x2 nf = 1/2


 4x
dx
4. u = 2x a2 = 25
Example 17 2
 25 du = 2dx a=5
1 5x
Use the special formulas to evaluate the = arctan +c Answer.
following integrands. 3 3
u2 = (x – 3)2 nf = 1
 3  x 2 dx
 x sec 1  3x dx  x 2  6x
u = (x – 3) a2 = 9
5 xdx
1. 8. 5.
du = dx a=3

  cos
2
2. 1  4 x 2 dx 9. (5 x  3) dx
 ( x 2  6x  9)  9 =  ( x  3) 2  9 + c
xdx xdx
=

 9  5x2 e
dx 2x
3. 10. cos 2 (e 2 x  1) dx
1 x3
= arctan +c Answer.
3 3
2  2x 
 4x  sec  3  5 dx
dx
4. 11.
2
 25

35
Let: u2 = (x – 1)2 nf = 1
 x2  2 x
dx
6.
u = (x – 1) a2 = 1 = (x2 – 1) arcsin (x2 – 1) + 1  ( x 2  1) 2 + c
, du = dx a=1
= (x2 – 1) arcsin (x2 – 1) + x 2  x 2 + c
 ( x2  2x  1)  1
dx
=
Answer.

( x  1)  1
14.
e
x 
arctan 1  2e x dx 
 ( x  1) 2  1
dx 1

 arctan1  2e 
= = ln +c
2(1) ( x  1)  1 x  e x dx
=

1 x Let u = 1 – 2e , du = –2ex dx, nf = –1/2


x
= ln +c Answer.
2 x2 1
=  (1 – 2exu arctan (1 – 2ex)
2
Let u2 = 2x2, u = 2 x, du = 3 dx – ½ ln|(1 – 2ex)2 + 1|
 5  2 x2
dx
7. 2
nf = 1, a = 5 , a = 5
Self-Assessment Test 8
1 5  2x
= ln +c Answer.
2 5 5  2x Evaluate the following integrals:

 x 2  11 
dx
5  9 x 2 dx
 x sec1  3x dx
1. 11.
5
8. 5 Let u = 1 – 3x
du = –15x dx
   17  9 x2  9  4x 2
dx dx
nf = –1/15 2. 12.
 sec 1  3x  x dx
5
=
1
=
 
ln |sin (1 – 3x5) + tan (1 – 3x5) + c dx
15 3. 16  25 x 2 dx 13.
Answer. 5  4 x2

 cos  x sec(x  sec(1  x) dx


2 2
9. (5 x  3) dx 4.  4) dx 14.

Let u = 5x – 3, du = 5 dx, nf = 1/5


  25  11x2
dx dx
5. 15.
1 1
= (5x – 3) + sin 2(5x – 3) + c Answer. 10  16 x 2
10 20

 3x 2  4  3x2 16
dx dx
6. 16.
e
2x 2 2x
10. cos (e  1) dx
Let u = e – 1, du = 2e2x dx, nf = 1/2
 sin  cos
2x
2 2
7. (5 x  2) dx 17. ( 2 x  5) dx
1 2x 1
(e – 1) + sin 2(e2x – 1) + c

= Answer.
 sec
2 dx
4 8 8. (2  3 x) dx 18.
19  49 x 2
2  2x
  2x 

3
11. sec   5  dx = tan   5 + c
   
  9x2  5
3 2 3 dx
9. 25  x 2 dx 19.
2x 2 3 Answer.
Let u =  5 , du = dx, nf =
3 3 2
10.  cos 2 x cot 2 x dx 
20. cot 2 3 x dx

sec2 ln x 
  sec ln x   x
1 2 dx
12. dx = Bonus Problem 3
2x 2
dx For P50 worth of cellphone load (@ P25 each),
Let u = ln x, du = dx, nf = 1
x give the correct answer.
1

 (3x
= tan (ln x) + c Answer. (2 sin 2 x  1)dx x 3 dx
2 1.
 1  sin 2 x
2.

 
2
 2) 2

2
13. arcsin x  1 dx

 (sin x  cos x) 2
cos 2 x dx
3.
Let u = x2 – 1, du = 2x dx, nf = 1/2

36
F.1.3.2 Integration Procedures Let u = x2, Let u = x
du = 2x dx du = dx
Aside from specific formulas listed in
Attachment G, integration of functions may be dv = cos 2x dx dv = sin 2x dx
undertaken by integration methods and 1 1
v = sin 2x v =  sin 2x
procedures. Again, we will just cover here the 2 2
most common and important ones which you
1 2  1 1 
will encounter in Part 2 of our course. These = x sin 2x –   x sin 2 x   sin 2 x dx 
are: 2  2 2 
1. Integration by Parts (I by P) 1 2 1 1
2. Integration of Rational Fractions by Partial
=
2
x sin 2x + x sin 2x –
2 2  sin 2 x dx
Fractions (IRF by PF) 1 2 1 1
= x sin 2x + x2sin 2x + cos 2x + c
3. Trigonometric Substitution (TS) 2 2 2
4. Algebraic Substitution (AS) Answer.

5. Powers of Sines and Cosines (PSC) 2.


e
x
sin x dx = –e–xcos x – e  x cos x dx

6. Powers of Tangents and Secants (PTS)
7. Powers of Cotangents and Cosecants Let u = e–x, Let u = e–x,
(PCC) du = –e–xdx du = –e–xdx
dv = sin x dx, dv = cos x dx,
F.1.3.2.1 Integration by Parts v = –cos x v = sin x

 
This method is ideally used to integrate product
of two functions. Its formula is = –e–xcos x –  e  x sin x  e  x sin x dx 
`IM-58:
e
x
sin x dx = –e–x cos x + e–x sin x

 u dv = uv –  u dv ; u = f(x), v = f(x) x
– e sin x dx

Suggested Procedure:

2 e  x sin x dx = –e–x cos x + e–x sin x + c
1. From the given integrand, identify u and dv.

e
x 1 –x
2. Formulate du by differentiation and v by sin x dx = e (sin x – cos x) + c
2
integration.
Answer.
3. Substitute to IM-58.

x e
1 2 2x
 xe
4. Repeat the integration process until the “” is 3. 2 2x 2x
dx = x e – dx
cleared. 2
5. Simplify your answer and affix the constant of
Let u = x2 du = 2x dx Let u = x du = dx
integration c.
1 2x 1
dv = e2x dx v= e dv = e2x dx v = e2x
Example 18 2 2
Integrate. 1 2 2x  1 2x 1 2x 
= x e –  xe  e dx  
x x 2 2 2 
2 2
1. cos 2 x dx 4. ln x dx


1 2 2x 1 2x 1 2 x
2.  e  x sin x dx 5.  sec3 3 x dx =
2
x e – xe +
2 2
e dx

1 2 2x 1 2x 1 2x
x e
2 2x
3. dx = x e – xe + e + c
2 2 2

 
Solution:
e2x
= 2x 2  2x  1 + c Answer.

1 2 4
1. x 2 cos 2 x dx =
2
x sin 2x –  x sin 2 x dx
37
x
1 3 1 N ( x)
4.
2
ln x dx = x ln x –  x 2 dx RF = (D-24)
3 3 D( x)

1 1 3 where:
Let u = ln x, du = dx dv = x2 dx, v = x
x 3 RF = rational fraction function of x
N(x) = numerator function of x
1 1 x3
= x 3 ln x – x 3 + c = (ln x – 1) + c D(x) = denominator function of x
3 9 9
Answer. Except for limited cases, direct integration of (D-
24) cannot be done due to very few applicable
 sec  sec3x sec
5. 3 2
3x dx = 3x dx integration formula (e.g., power formula and
natural logarithm formula), the reason why we
are resorting to integration procedures. Similar
Let u = sec 3x , du = 3 sec 3x tan 3x dx with numerical fractions, we can classify (D-24)
1 into two types:
dv = sec2 3x dx, v = tan 3 x
3
1. Proper rational fraction – when the degree
1 of N(x) ≺ degree of D(x), and
= sec3x tan 3x –  sec 3x tan 2 3x dx
3
2. Improper rational fraction – when the
1
= sec3 x tan 3 x –  sec 3x (sec2 3x - 1) dx degree of N(x) ≧ degree of D(x).
3
1 Important: In the case of improper rational
= sec3x tan 3x –  sec3 3x dx +  sec 3 x dx fraction, divide first N(x) by D(x) before
3
integrating.


1
2 sec3 3x dx = sec3x tan 3x +  sec 3 x dx Our task here is to transform (D-24) into their
3
corresponding partial fractions in order for us to
use the appropriate the integration formulas. We

1
2 sec3 3x dx = sec3x tan 3x will take up five cases here which are based on
3
the form of the denominators D(x):
1
+ ln (sec 3x + tan 3x) + c Case 1: Distinct linear factors
3
Case 2: Repeated linear factors

1
sec3 3x dx = [sec 3x tan 3x Case 3: Distinct quadratic factors
6
+ ln (sec 3x + tan 3x)] + c Answer. Case 4: Repeated quadratic factors
Case 5: Combination of factors
Self-Assessment Test 9 We will take this up one by one here.
Evaluate the following integrals.
Case 1: D(x)  distinct linear factors

 x  1 x
2 2x
1. 2 x 2  1 dx 4. e dx N ( x) A B
= +
(ax  b)( cx  d )...( ex  f ) ax  b cx  d
e x
x 2
2. sin(x  1) dx 5. ln x 2 dx
C
+…+
ex  f
x 
2
3. cos 2 x dx 6. xe x cos x dx
Case 2: D(x)  repeated linear factors
F.1.3.2.2 Integration of N ( x) A B C
Rational Fractions by Partial = + +…+
(ax  b) n ax  b (ax  b) 2
(ax  b) n
Fractions

Recall that in your Algebra course, you have Case 3: D(x)  distinct quadratic factors
encountered mathematical expressions N ( x)
containing numerator and denominator. These
are called rational fractions which are written in (ax2  bx  c)( dx2  ex  f )...( gx2  hx  i)
the form of

38
y3
Ax  B Cx  D
 (y  y 3  y dy
2
=  1) dy + (1-b)
= +
ax2  bx  c dx 2  ex  f
st
Ex  F The 1 term of (1-b) is already integrable.
+…+
gx2  hx  i y3
Case 4: D(x) contains repeated quadratic
 ( y 2  1) dy =
3
+ y + c1 (1-c)

factors: nd
Its 2 term, however, can only be evaluated by
N ( x) Ax  B IRF by PF, thus
= +
(ax 2  bx  c) n 2
ax  bx  c y3 y3
=  Case 1
y y3 y ( y  1)( y  1)
Cx  D
+…
(ax 2  bx  c) 2 y3 A B C
= + + (1-d)
y y3 y y 1 y 1
Case 5: D(x)  combination of any of the
four cases above. where A, B and C are constants to be
determined by algebraic procedures. Eliminating
the denominators in (1-b) by multiplying the
Example 19 equation with “y3 – y” we have:
Evaluate the following integrals. y + 3 = (y + 1)(y – 1)A + y(y – 1)B
+ y(y + 1)C (1-e)


y5  y  3
 x  2x  x  2
( x 2  5)dx
1. dy 2. We will evaluate the constants in (1-c) by three
3 2
y3  y methods:


( z  1) dz 1. By substituting a number that will eliminate
3.
3 2 a constant;
z  3z  3z  1
2. By equating coefficients of the same degree


(12 w 3  4w 2  3w  3) dw of variables; and
4.
w 4  3w 3  1 3. By combination of 1 and 2 above.


( z 5  2 z 3 ) dz Each method has advantage over the other,
5. depending on the type of rational fraction.
z 6  3z 4  3z 2  1
Method 1: By substitution

 x  4 x  6 x  8x  8x
( x 2  2 x  3) dx
6. a. Substituting y = 0 in (1-c):
5 4 3 2
0 + 3 = (0 + 1)(0 – 1)A + 0(0 – 1)B

x
(5 x 4  22 x 2  18 x  16) dx + 0(0 + 1)C
7.
5  4 x 4  6 x 3  8x 2  8x
3 = –A + 0B + 0C  A = –3
Solution: b. Substituting y = –1 in (1-c):
–1 + 3 = (–1 + 1)(–1 – 1)A + (–1)( –1 – 1)B


y5  y  3
1. dy (1-a) + (–1)( –1 + 1)C
y2 + 1
y3  y
y3 – y y5 – y – 3 2 = 0  A + 2B + 0  C  B=1
Eq. (1-a) is an improper y5 – y3 c. Substituting y = 1 in (1-c)
rational fraction, so we will
y3 1 + 3 = (1 + 1)(1 – 1)A + (1)(1 – 1)B
first divide the numerator
y3 – y – 3
by the denominator, thus + (1)(1 + 1)C
y+3
 
  y 2  1  y  3  dy 4 = 0  A + 0  B + 2C  C=2
=
 y 3  y 
 d. Substituting values to (1-b)

39
y3 3 1 2
= + + (1-f) Method 2: By equating coefficients of the same
y y3 y y 1 y 1
degree of “y”.
Note that while we cannot integrate the LHS of From (2-c):
(1-d), its RHS can be integrated using the “ln”
formula. Before we proceed with the integration, x2 + 0  x + 5 = (x2 + 3x + 2)A
let us first discuss the other two methods. + (x2 + x – 2)B + (x2 – 1)C (2-d)
Method 2: By equating coefficients of the same and using (2-d):
degree of “y”.
x0 : 5 = 2A – 2B – C (2-e)
a. Expanding (1-c) 1
x : 0 = 3A + B (2-f)
0  y2 + 1  y + 3 = (y2 – 1)A + (y2 – y)B x 2
: 1=A+B+C (2-g)
+ (y2 + y)C
Solving (2-e), (2-f) and (2-g) simultaneously
b. Coeff. of y0: 3 = –A  A = –3 gives A = 1, B = –3 and C = 3. (Same answers
with Method 1.) Substituting these values into
c. Coeff. of y1: 1 = –B + C  –B + C = 1 (2-b) gives
d. Coeff. of y2: 0 = A + B + C x2  5 1 3 3
= + +
0 = –3 + B + C  B + C = 3 ( x  1)( x  1)( x  2) x 1 x 1 x  2
Solving the equations in Items c and d gives (2-h)
B = 1 and C = 2. (Same answers with Method Finally, using (2-a)

 x  2x
1.). Going back to our problem ( x 2  5)dx  1 3 3 
3 2
x2
=   x  1  x  1  x  1  dx

y3 3 1 1 
dy =     dy
3
y y  y y  1 y  1  = ln (x – 1) – 3 ln (x + 1) + 3 ln (x + 1) + c
Answer.
= –3 ln y + ln (y + 1) + ln (y – 1) + c2

z
( z  1) dz
(1-g) 3. 3
 3z 2  3z  1
Substituting (1-c) and (1-g) into (1-b) gives


( z  1) dz z 1
 ( z  1)( z  1)( z  1) dz

y5  y  3 y3 = (3-a)
dy = + y –3 ln y ( z  1)3
y3  y 3
This is Case 2. Evaluating the constants:
+ ln (y + 1) + ln (y – 1) + c Answer.
z 1 A B C
= + + (3-b)


( x 2  5)dx x2  5 ( z  1) 3 z  1 ( z  1) 2 ( z  1) 3
2. = dx
x3  2 x 2  x  2 ( x  1)( x  1)( x  2)
(2-a) Multiplying (3-b) with (z – 1)3

This is Case 1. Evaluating the constants: z + 1 = (z – 1)2A + (z – 1)B + C (3-c)


2
x2  5 A B C or z + 1 = (z – 2z + 1)A + (z – 1)B + C (3-d)
= + +
( x  1)( x  1)( x  2) x 1 x  1 x  2
Method 3: By combination (We cannot use
(2-b) Method 1 alone since we have only one factor (z
– 1).
x2 + 5 = (x + 1)(x + 2)A + (x – 1)(x + 2)B
+ (x + 1)(x – 1)C (2-c) From (3-c) with z = 1 gives C = 2.
Using (3-d) with C = 2:
Method 1: By substitution
From (2-c) z + 1 = (z2 – 2z + 1)A + (z – 1)B + 2 (3-e)

x = 1 : 6 = (2)(3)A  A=1
2
z: 0=A  A=0
x = –1 : 6 = (–2)(1)B  B = –3 z1: 1 = –2A + B = –2(0) + B  B=1
x = –2 : 9 = (–1)(–3)C  C=3 Then, (3-b) becomes

40
z 1 1 2 Then (4-b) becomes
= + (3-e)
( z  1) 3
( z  1) 2
( z  1) 3 w 1 2w 2w  3
= + (4-i)
w  3w  1
4 3
w 1
2
w  3w  1
2
Important: In all instances, always check if the
RF (or LHS) is equal to the PF (or RHS) Plugging in (4-i) into (4-a) and integrating, we
before proceeding with the integration. have
( w  1) dw  2w 2 w  3 
 = 
Using (3-a)  dw
w4  3w3  1  w 2  1 ( w 2  3w  1) 
 1  
2 
 
( z  1)dz
=    dz = ln (w2 – 1) + ln (w2 – 3w + 1) + c Answer.
2
( z  1) 3  ( z  1) ( z  1)3 


( z 5  2 z 3 ) dz ( z 5  2 z 3 ) dz
=  [( z  1) 2  2( z  1) 3 ] dz 5. =
z 6  3z 4  3z 2  1
(z 2
 1) 3

 (z 1)( z 1)( z 1) dz


3
2z 1
( z  1) 1 2( z  1) 2 = (5-a)
= + +c 2 2 2
1 2
1 1 z This is Case 4. Evaluating the constants:
= – +c=– + c Answer.
z  1 ( z  1) 2
( z  1) 2 2z 3 1 Az  B Cz  D Ez  F
= + +
( z  1)
2 3
z 12
( z 2  1) 2 ( z 2  1) 3


(12 w 3  4w 2  3w  3) dw (5-b)
4. Multiplying (5-b) with (z2 + 1)3
w 4  3w 3  1
z5 – 2z3 = (z2 + 1)2(Az + B)

 (w
3 2
 12 w  4w  3x  3
= dw (4-a) + (z2 + 1)(Cz + D) + (Ez + F) (5-c)
2 2
 1)( w  3w  1)
or z5 – 2z3 = (z4 + 2z2 + 1)(Az + B)
This is Case 3. Evaluating the constants: + (z2 + 1)(Cz + D) + (Ez + F)
w 1 Aw  B Cw  D
= + (4-b) z5 – 2z3 = (z5 + 2z3 + z)A + (z4 + 2z2 + 1)B
4 3 2 2
w  3w  1 w 1 w  3w  1
+ (z3 + z)C + (z2 + 1)D + Ez + F (5-d)
Eliminating the denominators in (4-b) gives
We need six equations in order to evaluate the
–12w3 + 4w2 + 3w + 3 six constants in (5-d). Here, we will use Method
= (w2 – 3w + 1)(Aw + B) + (w2 – 1)(Cw + D) 2 (No choice!):

(4-c) z5: 1=A  A=1


Simplifying (4-C) 4
z: 0=B  B=0
–12w3 + 4w2 + 3w + 3 = (w3 – 3w2 + w)A 3
z: –2 = 2A + C  C = –4
+ (w2 – 3w + 1)B + (w3 – w)C + (w2 – 1)D 2
z: 0 = 2B + D  D=0
(4-d) 1
z: 0=A+C+E  E=3
We need four equations in order to evaluate the
four constants. Similar with Example 3, we will z0: 0=B+D+F  F=0
use Method 3 (No choice!). Then (5-b) becomes
Using (4-c) and (4-d) we have
z 5  2z 3 z 4z 3z
w = 1: –2 = –A – B (4-e) = – +
( z 2  1) 3 z 2 1 ( z 2  1) 2 ( z 2  1) 3
w0 : 3=B–D (4-f)
(4-e)
w3 : 4=A+C (4-g)
Plugging in (5-e) into (5-a) and integrating, we
w1 : 3=A+B–C–D (4-h)
have
Solving (4-e) to (4-h) simultaneously gives  z 
  dz
4z 3z
=  – +
( z  1) 
A=2 B=0 C=2 D = –3 2 3
 z 1 ( z  1)
2 2 2

41
 z 2 1 – 4  (z z 5  2z 3
z dz
= 2
 1) 2  z dz =
z

4z
+
3z
2
2
( z  1) 3 z 1 ( z 2  1) 2 ( z 2  1) 3
3
+ 3 ( z  1) 2
 z dz (5-e)
Plugging in (5-e) into (5-a) and integrating, we
1 ( z 2  1) 1 ( z 2  1)2 have
= ln (z2 + 1) – 4 +3 +c
2 1 2

z
( z 5  2 z 3 ) dz
1 4 3 6  3z 4  3z 2  1
= ln (z2 + 1) + – +c
2 z  1 2( z  1) 2
2 2


 z 4z 3z  dz
Answer. =  – +
 z 2  1 ( z 2  1) 2 ( z  1) 3
2 

x
( x 2  2 x  3) dz
6.


5  4 x 4  6 x 3  8x 2  8x
– 4 ( z 2  1) 2  z dz

z dz
=
2
z 1

 x(x  2) (x
( x 2  2 x  3) dx
=
2 2
 2) + 3 ( z 2  1) 3  z dz

This is Case 5. Evaluating the constants: 1 ( z 2  1) 1 ( z 2  1)2


= ln (z2 + 1) – 4 +3 +c
x 2  2x  3 A B C 2 1 2
= + +
x( x  2) ( x  2) 2 2 x x  2 ( x  2) 2 1 4 6
= ln (z2 + 1) + – +c
2 2
z  1 ( z  1) 2
2
Dx  E
+ (5-a) Answer.
x2  2

 x  4 x  6 x  8x  8x
(5 x 4  22 x 2  18 x  16 ) dx
This is Case 4. Evaluating the constants: 7. 5 4 3 2
2z  1
3
Az  B Cz  D Ez  F
= + +
 x  4 x  6 x  8x  8x
(5 x 4  22 x 2  18 x  16 ) dx
( z  1)
2 3
z 12
( z  1)
2 2
( z 2  1) 3 =
5 4 3 2
(5-b)
2 3
Multiplying (5-b) with (z + 1)


(5 x 4  22 x 2  18 x  16) dx
5 3
z – 2z = (z + 1) (Az + B) 2 2 = dx (7-a)
x( x  2)2 ( x 2  2)
+ (z2 + 1)(Cz + D) + (Ez + F) (5-c)
5 3
or z – 2z = (z + 2z + 1)(Az + B) 4 2 5 x 4  22 x 2  18 x  16 A B
= +
2 2
x( x  2) ( x  2) 2 x x2
+ (z + 1)(Cz + D) + (Ez + F)
z5 – 2z3 = (z5 + 2z3 + z)A + (z4 + 2z2 + 1)B C Dx  E
+ + (7-b)
3
+ (z + z)C + (z + 1)D + Ez + F 2
(5-d) ( x  2) 2 x2  2
We need six equations in order to evaluate the 5x4 + 22x2 – 18x – 16 = (x – 2)2(x2 + 2)A
six constants in (5-d). Here, we will use Method
+ x(x – 2)(x2 + 2)B + x(x2 + 2)C
2 (No choice!):
+ x(x – 2)2(Dx + E) (7-c)
z5: 1=A  A=1
z4: 0=B  B=0 5x4 + 22x2 – 18x – 24 =
(x4 – 4x3 + 6x2 – 8x + 8)A
z3: –2 = 2A + C  C = –4
+ (x4 – 2x3 + 2x2 – 4x)B
z2: 0 = 2B + D  D=0
+ (x3 + 2x)C + (x4 – 4x3 + 4x2)D
z1: 0=A+C+E  E=3
+ (x3 – 4x2 + 4x)E (7-d)
z0: 0=B+D+F  F=0
Using (7-c) and (7-d)
Then (5-b) becomes
x = 0 : –24 = 8A  A = –3
x = 2 : 108 = 12C  C=9

42
x1 : –18 = –8A – 4B + 2C + 4E u u
tan  = cot  =
–18 = –16 – 4B + 2 + 4E a2  u 2 a2  u 2
4B – 4E = 4  B–E=1 (7-e) a a
2
sec  = csc  =
x : 22 = 6A + 2B + 4D – 4E a2  u 2 u
22 = 12 + 2B + 4D – 4E
2B + 4D – 4E = 10 9  x2
cot  =
B + 2D – 2E = 5 (7-f) x
4
x :5=A+B+D Case 2: Integrand contains
5=2+B+D B+D=3 (7-g)
u 2  a 2 , where u = f(x) u 2  a2
Solving (7-e), (7-f) and (7-g) gives u
Let: u = a tan 
B=1 D=2 E=0 
du = a sec2  d a
Substituting values to (7-a), Triangle
 = arctan  
u


 3 1 9 2x  a
=  + + +  dx
 x x  2 ( x  2) 2 x 2
2 u a
sin  = cos  =
(7-h) u 2  a2 u 2  a2
9
= –3 ln x + ln (x – 2) – + ln (x2 + 1) + c u a
x2 tan  = cot  =  
Answer. a u

Self-Assessment Test 10 u 2  a2 u2  a2
sec  = csc  =
a u
Evaluate the following integrals.
Case 3: Integrand contains:

 
x dx x 4 1
1. dx 4. dx
x2  5x  6 x 3  9x u 2 9a2 x,2where u = f(x)
cot  = u
u 2  a2

 
x5 dx x
2. dx 5. Let: u = a sec  
x3  x x5  2x3  x
du = a sec  tan  d a
Triangle

 x
x4  4 x2  x  1 x2  4
 = arcsec  
u
3. dx 6. dx
x2  4 4 4 a

F.1.3.2.3 Trigonometric u2  a2 a
sin  = cos  =
Substitution u u

This method is used to evaluate integrands u 2  a2 a


tan  = cot  =
containing sum or difference of squares of a a
u 2  a2
variable and a constant. We have three cases
u u
here: sec  = csc  =
a
u 2  a2
Case 1: Integrand contains:

a 2  u 2 , where u = f(x) a Example 20


u
Let: u = a sin 

du = a cos  d a2  u 2
Evaluate the following integrals.

 = arcsin  
 
u
Triangle x 3 dx 4  9 x 2 dx
a 1. 4.
1 4x 2 x4
u a2 u2
sin  = cos  =
a a

43
  
9  x 2 dx x 2 dx x = 3 sin  , dx = 3 cos  d, 3
2. 5. x
x2 x2  4
3/ 2

9  x 2 = 3 cos 
9  x2


5 x 2  16 dx Substituting values:
x
3
3. x 2  3 dx 6.
x3
 
3 cos (3 cos  d ) cos2  d
= =
Solution: (3 sin  )2 sin 2 
Case 1:
 
= cot 2  d = (1  csc2  ) d


3
1. x dx
, u = 2x a = 1
1  4x 2 = ( + cot ) + c
Replacing values:
2x = sin  x = ½ sin  dx = ½ cos  d
 
 arcsin  x   9  x
2
1  4 x 2 = cos 
1  +c
=
    Answer.
3 3 x
Substituting values: 1  
2x
3  Case 2:
1  1
 sin   cos d 1  4 x2


2  2
x
= 3. 3 x 2  3 dx
, u=x a= 3
cos

 sin  3 tan ,
1 3 1
=  d = sin 2  sin d Let: x =
x2  3
16 16
dx = 3 sec  d
2
x

 (1  cos
1 2
=  )sin d
16 x2  3 = 3 sec  3
Substituting values:

1
= (sin  cos2  sin ) d
16
=  3 tan  3  3 sec  3 sec2  d 
1 1
=  (cos  cos3  ) + c
16 3

= 9 3 tan 3  sec3  d
1 1
=  cos (1  cos2  ) + c
16 3 
= 9 3 tan 2  sec2   tan  sec d

Replacing values:

= 9 3 (sec2   1) sec2   tan  sec d

= 
1  4x 2  1
 2 

1  3 1  4 x  + c
16   
= 9 3 (sec4   sec2  )  tan  sec d

1  4x 2  4 4 2   sec5  sec3  
=    x  +c = 9 3   +c
16 3 3   5 3 
 

= 
1  4x 2
12

1  x2 + c  =
3 3

sec3  3 sec2   5 + c 
5

( x 2  1) 1  4 x 2 Replacing values:
= +c Answer.
12 3 2 
3 3  x 2  3   
  x2  3  
= 3
   5 +c
5  3  

9  x 2 dx 3 
2. ,u=x a=3      
 
x2

44
=
1 2
 3/2

x  3 ( x 2  2) + c Answer. =

(2 sec )( 2 sec tan  d )
5 8 tan3 

1  tan 1  

tan 2   sec2  d = 
1

4. 4  9 x 2 dx = +c
, u = 3x a = 2 2
9x  4 2 2   1 
x4 3x
Let: 3x = 2 tan  ,  1 1 
=    = –½ cot  + c
2 2 2 2  tan  
x= tan , dx = sec2  d
3 3
Replacing values:

9 x 2  4 = 2 sec  = (–½)
2
+c =–
1
+ c Answer.
Substituting values: x2  4 x2  4

2 sec  2 sec2  d 



6. 5 x 2  16 dx
3 

,u= 5x a=4
=
4 x3
2 
 tan  
3  Let: 5 x = 4 sec  , 5x
5 x 2  16
4
x= sec  
  sin  cos 
27 sec3  d 27 1 cos3  d 5
= = 4
4 tan4  4 4 4
4
dx = sec  tan  d


27 cos d 27 5
= =  sin 4  cos d
4 sin 4  4
5x 2  16 = 4 tan 
27  sin 3   9 1  Substituting values:
= + c =    + c

4  3   4  sin 3  
4 tan   4 sec tan d 
= 
9
4
csc3  + c =
 4
5

 sec tan 
3

Replacing values: 5 
d
 sec2 tan
25
=
9 x  4
3 4
9  9 x 2  4  2 3
1
=    + c = +c
cos
4 
 
3x 12 x3 25 25
  = cos2  cot  d = cos2  d
4 4 sin


1 9x 2  4 3/ 2
(1  sin2  )

25
=  +c Answer. =  cos d
12 x3 4 sin 

(cos  sin 2  cos )



Case 3: 25
= d
4 sin 

 
2
5. x dx

25
2 3/ 2
,u=x a=2 = (cot   sin  cos ) d
x 4 4
x
Let: x = 2 sec  , x2  4 25  sin 2  
 = ln sin   +c
4  2 
dx = 2 sec  tan  d 2

x 2  4 = 2 tan  , (x2 – 4)3/2 = 8 tan3  =


25
8

2 ln sin   sin 2  + c 
Substituting values:
Replacing values:

45
 2

 
x3dx ln 2 x dx
25  5 x 2  16  5 x 2  16   2. 5.
2 ln  x ln x 1
8   +c
=
 5 x  5 x   x 2 1
 
 
e 2x  2dx
  
2
 
2 3.  e x 1
25   5 x 2  16 
2
 5 x  16  
ln   +c
8     
=
5x   5x  
      Solution:

25   5 x 2  16  5 x 2  16   

x  1 dx Let: x =u
= ln  +c Answer. 1.
8   5x 2  2 
x 1
x = u2
   5 x  dx = 2u du
Substituting values:
Self-Assessment Test 11
u  1 2u du u2  u du
Evaluate the following integrals. =
 u 1
= 2
 u 1


x2

2 2
1. x 9  16 x dx 4. dx u+2

 2 
1  x2 = 2 u  2   du u – 1 u2 + u
 u 1 
u2 – u


4  x2  u2 
2.
 x 2 9  4x 2 5.
x
dx = 2  2u  2 ln (u  1) 
 2



2u
2u – 2


dx 2
x
2
3. 4 x 2  25 6. dx Back substituting values:
x2 x2  4
x 
= 2  2 x  2 ln ( x  1)  Answer.
F.1.3.2.4 Algebraic Substitution  2 

As the name suggests, this method uses x2  1 = u



x3dx Let:
transfer of variable in order to transform the 2.
x2 = u 2 + 1
integrand into an integrable form. In some x 2 1 x dx = u du
instances, terms containing radicals are
eliminated by raising to powers or a group of Substituting values:
variables replaced with a single variable.
1.

 
x 2  x dx (u 2  1)(u du)
Suggested Procedure: = =
x2  1 u
1. Formulate an appropriate substitution, z =

 
f(x), where f(x) is an expression that will u3

u
transform the integrand into integrable = (u 2  1) du =  u + c = u2  3 + c
form. 3 3

2. Substitute Step 1 into the problem. Back substituting values:


3. Integrate Step 2 using applicable formula
or method. =
1 2
3
 
x  2 x 2 1 + c Answer.
4. Backsubstitute for x; and
Do This: Solve Item 2 using TS.
5. Simplify your answer.

  Let: ex = u e2x = u2


3. e2 x  2 dx ln ex = ln u x = ln u
Example 21
ex  1 du
dx =
Evaluate the following integrals. u

 u
u 2  2 du u2  2
  = du

 
x  1 dx sin 2 x sin x  1 dx u 1 u 2 u
1. 4.
x 1 sin x  1

46

 u2  8  sin x  1 
= 1  du 1 + arctan  + c Answer.
 u2  u   
u2 + u u2 –2 2  2 


u2 u2 + u
=  du – du


u (u  1) –u – 2 ln 2 x dx
5. Let: ln x  1 = u
nd x ln x 1 ln x + 1 = u2
Using IRF by PF on the 2 term
ln x = u2 – 1


ln 2 x dx
u2 A B 2 1 = ln2 x = (u2 – 1)2
u (u  1)
= +
u u 1
= +
u u 1 ln x 1 x dx
= 2u du
x

u + 2 = (u + 1)A + uB (u 2  1)2
= 2u du
u
u=0 : A=2
u = –1 : B = –1  
= 2 (u 2  1) 2 du = 2 (u 4  2u 2  1) du


 2 du du 

= du – 
 u
 
u 1 
 u 5 2u 3
= 2 

u + c
 5 3 
= u – 2 ln u + ln (u + 1) + c  

Back substituting values:


x x x
=
2
15

u 3u 4  10u 2  15 + c 
= e – 2 ln e + ln (e + 1) + c
Back substituting values:
= ex – 2x + ln (ex + 1) + c Answer.
=
2

ln x  1 3ln x  12  10ln x  1  15 + c


sin 2 x sin x  1 dx 15
4. Let: sin x  1 = u
sin x  1
sin x – 1 = u2
sin x = u2 + 1
=
2
 
9 ln 2 x  8 ln x  14 ln x  1 + c

 sin x 1
2 sin x cos x sin x  1 dx 15
= cos x dx = 2u du Answer.

Self-Assessment Test 12
 sin x 1
sin x sin x  1  cos xdx
= 2

 
u2 – 1 x3 2 x  1 dx
1. dx 5.


(u 2  1)u  2u du 2
u +2 u +u 4 2
x2  1 x
= 2
2
(u  1)  1 u4 + 2u2


e2 x
–u2 2.  1  cos x dx 6. dx

 u 2
(u 4  u 2 ) du –u2 – 2
= 4 ex 1
2 2

 ln
e2 x

dx

  u  2 
 2  3. 7. dx
2 2
= 4  u  1  du x 3 x x 1
2

x  sin x 
x dx cos x dx

 
4. 8.


 du  x 1
= 4 u  1 du + 8  
2 sin x
 u2  2 
 u3  Bonus Problem 4
8  u 
= 4 u + arctan  + c

 3    For P50 worth of cellphone load (@ P25 each),
  2 2
give the correct answer.
4

u u2  3 +
8
  u 
arctan  + c

= 
 1  sin x
dx dx
3 2  2 1. 2.
x 1
Back substituting values:


6 x dx
3.
= sin x  4 sin x  1
4 3x x
3

47
F.1.3.2.5 Powers of Sines Example 22
and Cosines Evaluate the following integrals.
This method is used to evaluate integrands
sinx / 3
which can be reduced into the product of sine
and cosine in the form of: 
1. sin 2 3 x dx 12.
 cos4 x / 3
dx

 sin
m
u cosn u du
  sec2 2 x
(TS-1) sin 2 x dx
2. sin 4 2 x dx 13.

where u = f(x), m and n = any number

 
cos x
3. cos2 5 x dx 14. dx
Suggested Procedure: cot x

 cos  tan x csc


Step 1. Transform the integrand into TS-1 4. 6
x dx 15. 3 2
x dx
using (whichever is applicable):
sin2 u + cos2 u = 1
 sin  sin
5
(TS-2) 5. 3
4 x dx 16. 2 x cos3 2 x dx
sin2 u = ½ (1 – cos 2u) (TS-3)
 sin  sin
5 5/ 2
6. 3 x dx 17. x cot5 x dx
cos2 u = ½ (1 + cos 2u) (TS-4)
sin 2u = 2 sin u cos u (TS-5) 3


cos2 5 x
Step 2. Evaluate the integrand using any of 
7. cos3 2 x dx 18.
csc3 5 x
dx
the following formulas:
5x 
 cos  sin
5
d du 8.   dx 19. 3 x cos4 3 x dx
D-39: (sin u) = cos u 2
dx dx

D-40:
d
dx
(cos u) = –sin u
du
dx
9.  sin
5

2 x  cos 2 x dx 20. sin 2 3 x cos2 3 x dx

 du 
cos3 x
I-1:
 sin
4
=u+c 10. dx 21. x cos2 x dx
csc3 x
u n 1
 u du = n  1 + c
n
I-4:
11.  cot x 3 sin x dx
I-31:  sin u du = –cos u + c
Solution:
I-32:  cos u du = sin u + c
Note: The following problems will be solved
Step 3. Simplify your answer. straightforwardly. Your Sir Anacta, therefore,
advises you to familiarize first the formulas and
the suggested procedure before proceeding with
We will discuss nine cases here.
the solutions. If you have concerns, just post the
same in our online class or text him.
Table 2. The Scope of POSC

Case m n Let u Case 1: Here, (TS-1) becomes


1
2
even
0
0
even
-
-  sin m u du   sin2 u m/ 2 du
3 odd 0 sin function
 sin
1. 2
3 x dx
4 0 odd cos function
5 any 1 sin function Using (TS-3), I1, I31 and simplifying:
6 1 any cos function
 sin
1
1  cos 6x dx
2
2
3 x dx =
7 any odd > 1 sin function
8 odd > 1 any cos function
1 1 
9 even even either =  x  sin 6 x  + c
2 6 

48
=
x

2 12
1
sin 6 x + c Answer.
 cos
n
u du   cos
2
u n/ 2 du
 sin  cos
2. 4 2
2 x dx 3. 5 x dx

Note: We will use: Using (TS-4), I1, I31 and simplifying:


amn = (am)n, (TS-3), (ab)n = anbn and
2 2 2  cos
2 1
1  cos 10 x dx
5 x dx =

(a – b) = a – 2ab + b . 2

 
1 1
Then we will have: = dx + cos 10 x dx
2 2

 sin 2x dx
2
 sin
4 2 where:
2 x dx =
2a.  dx = x + c  Use I1
2
1 
  2 1  cos 4 x  dx = 4  1  cos 4 x2 dx
1
=
  2b.  cos 10 x dx  Use I32

=
1
2  1  2 cos
2
2 x  cos4 2 x dx =
1
10
sin10 x + c

Substituting values:
 
1

1
= dx – cos2 2 x dx + cos4 2 x dx
1 1 

1
2 2 cos2 5 x dx = x +  sin 10 x 
where: 2 2  10 
x 1
2a.  dx = x + c  Use I1 = – sin 10 x + c Answer.
2 20

 cos  cos
2 6
2b. 2 x dx  Use (TS-4), I1 and I32 4. x dx

 1  cos 4x dx 
1 1

1 Note: For this problem, we will use:
= = dx + cos 4 x dx
2 2 2
amn = (am)n, (TS-3), (ab)n = anbn and
1 1 sin 4 x
= x +c
2 2 4 (a – b)3 = a3 – 3a2b + 3ab2 – b3
x 1 Then we will have:
= + sin 4 x + c (2-c)
 
2 8
 cos
3
6
x dx =  cos2 x dx
 cos 2 x dx  See Case 2
4
2c.
3


1 
 2 1  cos 2 x  dx = 8  1  cos 2 x  dx
1 3
=
=
3x 1
 sin 4 x 
1
sin 8 x + c  

 1  3 cos 2x  3 cos 
16 16 128
1 2
= 2 x  cos3 2 x dx
Substituting values and simplifying: 8
x x 1 
  
–  + sin 4 x  1 3 3
= = dx + cos 2 x dx + cos2 2 x dx
2  2 8  8 8 8

1  3x 1  1
8
1 3
+   sin 4 x  sin 8x  + c + cos 2 x dx
2  16 16 128 
where:
3x 3 1
= – sin 4 x + sin 8 x + c
32 32 256 4a.  dx = x + c  Use 2a
Answer.


Case 2: 1
4b. cos 2 x dx = sin 2 x + c  See 2b
2
Here, (TS-1) becomes


4c. cos2 2 x dx  See Item 3

49
=
x 1
+ sin 4 x + c  
= 1  2 cos 2 3 x  cos 4 3 x sin 3 x dx
2 8

 sin 3x dx – 2  cos 3x  sin 3x dx


2
=
 
4d. cos3 2 x dx = (cos 2 2 x)  cos 2 xdx
+  cos4 3 x  sin 3 x dx

= (1  sin 2 2 x )  cos 2 xdx
1 2 cos3 3 x 1 cos5 3 x
=  cos 3x   +c
 
2
= cos 2 x dx – sin 2 x  cos 2 x dx 3 3 3 3 5
1 1 1
where: =  sin 3x  cos3 3x  cos5 3x + c
3 9 15
 sin
2
4e. 2 x  cos 2 x dx  See Case 5 Answer.
Case 4:
1 sin3 2 x 1 Here, (TS-1) becomes
= + c = sin3 2 x + c
2 3 6
n 1
 cos  cos
n
u du  u  cos u du
Substituting values and simplifying:

 cos 
3
x 31  3 x 1  2 x dx = (cos 2 2 x)  cos 2 xdx
 cos
6 7.
x dx = +  sin 2 x  +   sin 4 x 
8 82  8  2 8 
Using (TS-2), I32, I4 and simplifying:
11 1 
+  sin 2 x  sin3 2 x  + c
82 6  
= (1  sin 2 2 x )  cos 2 xdx

 cos6 x dx =
5x 1
16
+ sin 2x +
4
3
64
sin 4 x 
= (cos 2 x  sin 2 2 x  cos 2 x) dx

1 1 sin 3 2 x

1
sin3 2 x + c Answer. = sin 2 x  +c
2 2 3
48
1 1
Case 3: = sin 2 x  sin 3 2 x + c Answer.
2 6
Here, (TS-1) becomes
2


x   2x  x 
 sin
m
u du   sin
m 1
u  sin u du 8.
 cos5   dx =
2
cos 
 2
  cos  dx
 2

 sin  Using (TS-2), (a – b)2, I32, I4 and simplifying:


3
5. 4 x dx = sin 2 4 x  sin 4 x dx
2


 2  x  x 
Using (TS-2), I31, I4 and simplifying: = 1  sin    cos  dx
  2  2
 
= 1  cos 2 4 x  sin 4 x dx
 x  x  x 

= 1  2 sin 2    sin 4    cos  dx
    2  2  2
2
= sin 4 x dx – cos 4 x  sin 4 x dx
x  2x  x 
1 1   cos3 4 x 
=  cos 4 x   +c 
= cos 
2
dx – 2 sin    cos  dx
  
2 2
4 4  3 

x  x 
1 1 +  sin4    cos  dx
= cos 4 x  cos3 4 x + c Answer.  
2 2
4 12
x  2 x  2 x 

 sin 
2 = 2 sin   – 2  sin 3   + sin 5   + c
 sin 2 3 2 5 2
5 2
6. 3 x dx = 3x sin 3x dx

Using (TS-2), (a – b)2, I31, I4 and simplifying: x  4 x  2 x 


= 2 sin   – sin3   + sin 5   + c
2 3 2 5 2

= 1  cos2 3x sin 3x dx 2 Answer.

50
Case 5: 1 / 2
=  cos x  sin xdx = 2 cos x + c
 sin
m
u  cos u du (TS-1) Answer.

 cos x
sin3 x
 sin
5

9. 2 x  cos 2 x dx 1
15. tan3 x csc2 x dx =  dx
3
sin 2 x
Let: u = sin 2x; du = 2 cos 2x, nf = ½
cos- 2 x
=
1 sin 2 x
=
61
sin 6 2 x Answer.
=  cos-3 x  sin x dx =
2
+c
2 6 12
1 1 1
=  + c =  sec2 x + c Answer.
 dx =  sin 3x
cos 3x 1/ 2
10.  cos3x dx 2 cos 2 x 2
csc 3x
1 Case 7:
Let: u = sin 3x; du = 3 cos 3x, nf =
3

=
1 sin 3 / 2 3 x
+c =
2
sin 3 3x + c Answer. 16.
 sin
5

2 x cos3 2 x dx = cos3 2 x sin 2 2 x dx  2
3 3/ 2 9

 
= cos3 2 x 1  cos 2 2 x2  sin 2x dx
  sin x 1/ 3 dx
cos x
11. cot x 3 sin x dx =
=  cos 2 x 1  2 cos 2 x  cos 2 x  sin 2 x dx
sin x
3 2 4
sin x  1/ 3
 sin x 
2 / 3  cos x dx
=  cos 2 x  2 cos 2 x  cos 2 x  sin 2 x dx
= = +c
1/ 3 3 5 7

= 3 3 sin x + c Answer. Let: u = sin 2x; du = 2 cos 2x, nf = ½

Case 6: 1  cos4 2 x cos6 2 x sin 8 2 x 


=    +c
2  4 3 8 
sinx / 3
12.
 cos x / 3
4
dx =  cos4 x / 3  sinx / 3 dx Answer.

Do This: Prove that the answers of Items 12


x 1 and 14 are the same.
Let: u = , du = dx , nf = 3
3 3
 cos5 x 
= 3
cos3 ( x / 3)
3
+c = 
3
1
+c
17.
 
sin5 / 2 x cot5 x dx = sin5 / 2 x   dx
 sin5 x 
 
cos ( x / 3)

 sin x cos x dx
sin5 / 2 x 5
=  sec3 ( x / 3) + c Answer. =
5

 sec2 2 x
sin 2 x dx
13. =  2 sin 2 x cos 2x cos2 2 x dx = sin 5 / 2 x  cos5 x dx


= 2 cos3 2 x  sin 2 x dx =
 sin  2
5 / 2
x cos2 x  cos x dx

= sin5 / 2 x 1  sin2 x   cos x dx


Let: u = sin 2x; du = 2 cos 2x, nf = ½

2
 1 cos4 2 x 
= 2  + c = 1 cos4 2 x + c

 sin x1  2 sin x  sin xcos x dx


2 4  4
  = 5 / 2 2 4
Answer.

= sin 5 / 2 x  2 sin 1/ 2 x  sin x  cos x dx


14.

cos x
cot x
dx =  tan x cos1/ 2 x dx  3/ 2

2 2
= sin 3 / 2 x  4 sin 1 / 2 x  sin 5 / 2 x + c

sin x
= cos1 / 2 x dx =  cos1/ 2 x  sin x dx 3 5
cos x
Answer.

51
Case 8: 2


1
=   1  cos 12 x 2 dx
3 2


cos2 5 x

 1  2 cos12 x  cos 12 xdx
18. dx = cos2 / 3 5 x sin3 5 x dx
csc3 5 x 1 2
=
 
4

= cos2 / 3 5 x sin 2 5 x sin 5 x dx
  
1 1 1
= dx + cos12 x dx + cos2 12 x dx
=  cos2 / 3 5 x 1  cos2 5 x sin 5 x dx
4 2 4

   1  cos 24x dx
1 1 1
=  cos2 / 3 5 x  cos8 / 3 5 x  sin 5 x dx
= dx + cos12 x dx +
4 2 8

 
1 1

1
1 cos5 / 3 5 x  1 cos11 / 3 5 x  =
4
dx +
2
cos12 x dx +
8
dx
=   +c
5 5/3  5 11 / 3 
 

1
+ cos 24 x dx
3 3 8
= cos5 / 3 5x  cos11 / 3 5x + c

1
 
25 55 3 1
= dx + cos12 x dx + cos 24 x dx
Answer. 8 2 8
 sin
19. 5 4
3 x cos 3 x dx
3x 1 1  1 1 
= +  sin 12 x  +  sin 24 x 

  2
= cos4 3x sin 2 3x sin 3x dx
8
3x 1
2  12  8  24
1

=  sin 12 x + cos 24 x + c (20-d)


= cos4 3x1  cos2 3x  sin 3x dx
8 24 192

2

Substituting (20-b) and (20-c) to (20-a)


= cos4 3x1  2 cos2 3x  cos4 3x sin 3x dx
 =
5x 1
 sin 12 x +
1
cos 24 x + c
= cos4 3x  2 cos6 3x  cos8 3x  sin 3x dx
8 24 192
 Answer.
 sin x cos x dx
4 2
21.
1 cos5 3 x 2 cos7 3 x 1 cos9 3 x
=  
sin 2 x  cos2 x dx
+c


3 5 3 7 3 9 2
=
1 2 1
= cos5 3x  cos7 3x  cos9 3x + c 2
1  1 
=   1  cos 2 x   1  cos 2 x  dx
15 21 27
Answer. 2  2 
Case 9:
 1  cos 2x
=
1 2 1  cos 2 x  dx
8
 sin
2
20. 3 x cos2 3 x dx (20-a)
=
1
 1  2 cos 2x  cos
2 
2 x 1  cos 2 x  dx

 1 cos 6x 1 cos 6xdx


1 2 2
8
=

 1  2 cos 2x  cos
4 1 2
= 2x
=
1
4 
1
 1
1  cos 6 x dx =  dx –  cos4 6 x dx
4
4 4
8
 cos 2 x  2 cos2 2 x  cos4 2 x dx 
where:
(20-b)
=
1
8  1  cos 2x  cos
2
2 x  cos4 2 x dx 
 dx =x+c (20-c) (a) (b) (c) (d) (21-a)

 
where:
2
 cos cos2 6 x dx
4
6 x dx =
a.  dx = x + c
2


1  1
=  2 1  cos 12 x  dx
 

b. cos 2 x dx =
2
sin 2x+ c

52

x 1
c. cos2 2 x dx = + sin 4 x + c Step 2. Evaluate the integrand using any
2 8 of the following formulas:

 cos
4 3x 1 1 D-41: d (tan u) = sec2 u du
d. 2 x dx =  sin 4 x  sin 8x + c
8 16 64
D-43: d (sec u) = tan u sec u du
Substituting the four values above to (21-a) and u n 1
simplifying: I-4:  u n du =
n 1
+c
1  15 x 1 1 1 
=   sin 2 x  sin 4 x  sin 8 x  I-33:  tan u du = –ln |cos u| + c
8 8 2 16 64 
1  15 x 1 1  I-35:  sec u du = ln |sin u + tan u| + c
=   sin 2 x  sin 4 x  sin 8 x 
16  4 8 32 
I-41:  sec2 u du = tan u + c
Answer.
I-52:  tan u sec u du = sec u + c
Self-Assessment Test 13
Solve the following: Step 3. Simplify your answer.

 x

1. cos4   dx
3 
6. tan 2 x  cos5 2 x dx We will discuss six cases here.

Table 2. The Scope of POTS


sin x  cos x 2 dx
 
5
2. sin x dx 7. Case m n Let u
sin 2 x
1 even 0 -


cos3 x
3.  sin x  cos x 3 dx 8. dx 2 0 even -
3
csc2 x 3 odd 0 tan function


cos x 4 0 odd -

4. csc4 3 x  cos 3 x dx 9.
csc3 x
dx
5 odd any tan fucntion
6 any even tan function

sin 2 x

3 3/ 2
5. sin x  cos x dx 10. dx
sec4 x
Example 23
Bonus Problem 5 Evaluate the following integrals.

 tan  sec3 2 x tan 2 x dx


2
1. 3 x dx 7.

1. sin 6 2 x dx 
2. sin 7 x dx

 cos4 3x dx
sin 3 x
 tan
4
2. 2 x dx 8.
F.1.3.2.6 Powers of Tangents
and Secants
Solution: 2 x 
 sec    dx  sec
5
3. 9. 2 x tan 3 2 x dx
This method is used to evaluate the product of
tangent and secant in the form of:


sec2 7 x
 tan
m
u sec n u du (PTS-1)
4.  sec6 x dx 10.
cot3 7 x
dx

 tan
3 2x
where u = f(x), m and n = any number
 tan
4/3
5. dx 11. x sec4 x dx
3
Suggested Procedure:
 tan
5
6. 4 x dx
Step 1. Transform the integrand into
PTS-1 using (if necessary) Solution:
2 2
sec u = 1 + tan u (PTS-2) Note: The following problems will be solved
straightforwardly. Your Sir Anacta, therefore,

53
advises you to familiarize first the formulas and 2 1
= tan 2 x  tan3 x  tan5 x + c Answer.
the suggested procedure before proceeding with 3 5
the solutions. If you have concerns, just text him Case 3:
or post the same in our online class.

 tan  tan 3 tan


5. 3 2x 2x 2 2x
Case 1: dx = dx
3 3

 tan  sec 
2 2x  
1. 2
3x  1 dx
 tan 3  sec
3 x dx = 2 2x
=  1 dx
3 
1
= tan 3x – x + c Answer.


3  2x 2x 2x 
=  tan sec2  tan  dx
 
2  3 3 3 
 tan4 2 x dx =  tan 2 x
2
2. dx
3  2x  3  2x 
= tan 2    ln  cos  + c Answer.
=  sec
2 2 x  1 dx2 4  
3 2  3 

  tan 
 sec
4 2
6. 5
4 x dx = tan3 4 x  tan2 4 xdx
= 2 x  2 sec 2 x  1 dx (2-a)
(c) (b) (a) =  tan 3 4 x (sec2 4 x  1) dx
where:
a.  dx = x + c
=  tan
3
4 x sec2 4 x  tan 3 4 x dx 

b. sec2 2 x dx =
1
tan 2x + c 
= tan3 4 x  sec2 4 x dx –  tan 3 4 x dx
2

 sec4 2 x dx = sec 2 x sec 2 x dx


2 2 
= tan3 4 x  sec2 4 x dx

 
c.
–  tan 4 x sec2 4 x  1 dx
=  1  tan 2 2 x  sec2 2 x dx

=  sec 2 x  tan 2 x  sec 2 x dx


2 2 2

= tan3 4 x  sec2 4 x dx

–  tan 4 x  sec2 4 x dx –  tan 4 x dx


(b) (d)
1 tan 3 2 x 1 tan4 4 x 1 tan2 4 x 1
d.  tan 2 2 x  sec2 2 x dx = +c =   ln (cos 4 x) + c
2 3 4 4 4 2 4
1 1 1 1
= tan 3 2 x + c = tan4 4 x  tan2 4 x  ln (cos 4 x) + c
6 16 8 4
Substituting the four values above to (2-a) and Answer.
simplifying: Case 4:
1 1 Note: Refer to Section F.4.2.1 (Integration by
= tan 3 2 x – tan 2x + x + c Answer.
6 2 Parts).
Case 5:
Case 2:

3. 2 x 
 sec    dx =  tan 2x + c Answer. 7.
 
sec3 2 x tan 2 x dx = sec3 / 2 2 x tan 2 x dx


 sec x sec
2 = sec1 / 2 2 x  sec 2 x tan 2 x dx
 sec
4. 6
x dx = 2 2
x dx
1 sec3 / 2 2 x 1


= 1  tan2 x sec2 x dx 2 =
2 3/ 2
+ c = sec3 / 2 2 x + c
3

 
1
= 1  2 tan2 x  tan4 x  sec2 x dx
= sec3 2 x + c Answer.
3

54
 cos4 3x dx =  cos3 3x cos 3x dx   cot5 x cos2 x
sin 3 x sin 3 x dx
8. 1. tan6 3 x dx 4.


tan4 x

2. sec4 x dx 5. dx

 1 sin 3x 
=    dx cos4 x
3
 cos 3x cos 3x 

 cot x dx
sec2 x
3. 
6. tan 4 x sec4 x dx
 
5
= sec3 3 x tan 3 x dx = sec2 3 x  sec 3 x tan 3 x dx

1
= sec3 3x + c Answer. F.1.3.2.7 Powers of
9 Cotangents and Cosecants
or simply:
This method is used to evaluate the product of
1 cos3 3 x
 cos4 3 x  sin 3 x dx = +c cotangent and cosecant in the form of:
3 3
 cot
m
u cscnu du (PCC-1)
1
= sec3 3x + c Answer.
9 where u = f(x), m and n = any number

 sec
5
9. 2 x tan 3 2 x dx Suggested Procedure:


= sec4 2 x tan2 2 x  sec 2 x tan 2 x dx Step 1. Transform the integrand into
PTS-1 using (if necessary)

  
= sec4 2 x sec2 2 x  1  sec 2 x tan 2 x dx csc2 u = 1 + cot2 u (PTS-2)

 sec 2 x  sec 2 x  sec 2 x tan 2 x dx


Step 2. Evaluate the integrand using any
6 4
= of the following formulas:

1 1 D-42: d (cot u) = –csc2 u du


= sec7 2 x  sec5 2 x + c Answer.
14 10 D-44: d (csc u) = cot u csc u du
u n 1
 u du =
n
Case 6: I-4: +c
n 1
 cot u du

sec2 7 x I-33: = ln |sin u| + c
 tan 7 x  sec2 7 x dx
10. 3
dx =
cot3 7 x
4
I-35:  cscu du = ln |csc u – cot u| + c
1 tan 7 x 1
 csc u du = –cot u + c
2
= = tan 4 7 x + c Answer. I-41:
7 4 28
I-52:  cot u csc u du = –csc u + c
 tan
11. 4/3 4
x sec x dx
Step 3. Simplify your answer.


= tan4 / 3 x sec2 x  sec2 x dx

 
Note: The approach of PCC is similar with the

= tan4 / 3 x 1  tan2 x  sec2 x dx PTS.

=  tan
4/3 
x  tan10 / 3 x  sec2 x dx
Example 24

Evaluate the following integrals.


tan 7 / 3 tan13 / 3 x
x
 csc  cot
= +c 5
7/3 13 / 3 1. 4
2 x dx 4. x dx
3 3
tan 7 / 3 x  tan13 / 3 x + c
  cot
Answer. 3
=
7 13 2. csc2 x cot x dx 5. 3
4 x csc1 / 3 4 x dx

 sin 3x dx
Self-Assessment Test 14 cos2 3 x
3.
4
Evaluate the following integrands.

55
Solution:
 cot x  csc x dx
3 2
=

 csc  csc –  cot x  (csc2 x  1) dx


4
1. 2 x dx = 2
2 x  csc2 x dx


= 1  cot2 2 x csc2 2 x dx  =  cot x  csc x dx
3 2

–  cot x csc2 x dx +  cot x dx



= csc2 2 xdx +  cot 2 2 x csc2 2 xdx

1 1 cot 3 2 x  1 cot 2 x 
= cot 2 x  +c 1 cot 4 x   +
2 2 3 =  –
 4   2 

=
1
6

cot 2 x 3  cot 2 2 x + c  1
ln (sin x) + c

=
1
6

cot 2 x csc2 2 x  2 + c  Answer.
= 
cot 4 x cot 2 x
+ +
1
ln (sin x) + c
4 2 


Answer.

3
2. csc2 x cot x dx = csc2 / 3 x cot x dx
 cot
3
5. 4 x csc1 / 3 4 x dx
= csc1 / 3 x  csc x cot x dx
 =  cot
2
4 x csc2 / 3 4 x  csc 4 x cot 4 x dx
1 csc2 / 3 x 3
= + c =  csc2 / 3 x + c 4 x  1) csc2 / 3 4 x  csc 4 x cot 4 x dx
 (csc
2
2 2/3 4 =

33
= csc2 x + c Answer. =  (csc
4/3
4 x  csc2 / 3 4 x)  csc 4 x cot 4 x dx
4
1 csc7 / 3 4 x  1 csc1 / 3 4 x 
 sin
cos2 3x =   
3. dx 4 7/3  4 1/ 3 
4 3x  

csc7 / 3 4 x  + csc1 / 3 4 x  + c
3 3

 sin 3x sin 3x dx =  cot 3x  csc 3x dx


cos2 3 x 1 2 2 = 
= 28 4
2 2
Answer.
3
1 cot 3 x
=  +c
3 3 Self-Assessment Test 15

 cot  cot 
5 3
x dx = xt  cot2 x dx

4. dx
1. cot4 2 x dx 4.
cot5 x sin 2 x
 cot
3 2
= x  (csc x  1) dx

2. csc4 3 x dx  cot
3
5. x csc5 x dx

 cot 
3
= x  csc2 x dx – cot 3 x dx

3. cot3 x csc4 x dx

 cot 
3 2 2
= x  csc x dx – cot x  cot xdx

FINAL EVALUATION (POST-TEST)

Test 1. Given the function: Group 5: 2x4 – 7x2 – 2y + 5 = 0


Group 1: x4 – 6x2 – 2y + 5 = 0 REQUIRED:
Group 2: x4 – 5x2 + 2y + 4 = 0 1. Locate the x-intercepts;
4
Group 3: 2x – 6x – y + 4 = 0 2 2. Locate the y-intercept;
3. Locate the maximum and minimum points;
Group 4: x4 – 7x2 + y + 6 = 0 4. Locate the inflection point;
56
5. Find the slope at point (2, 0); 3. e2y – xey + 2x = 0
6. Find the slope at point (0, 2); and 4. eysin x – xey = 2
7. Show the screenshot of the graph.
5. ln(xy) – y ln x = 1
Test 2. Find the first derivative in derivative form
and its value at indicated point. Group 2:
1. x2 – 2xy – y2 + 2y + 5 = 0
Group 1:
2. x tan y – y sin x + x = 1
1. y = 3
x 2
x –x
4. y = (e + x) 2
3. e–2y – yex + 2y = 0

x2 4. eycos x – yex = 2


–x
2. y = 5. y = xe + x sin x
x 1 5. ln(xy) – x ln y = 1
3. y = (x – 1)2 (x + 2)3 Group 3:
1. x2 – 3xy – 5x – 3y + 1 = 0
Group 2:
2. x cos y – y tan x + y = 1
1. y = 3 x x 4. y = (ex – x)2 3. xe2y – yex + 2x = 0
x2 4. eytan x – xey = 2
2. y = 5. y = xex + x cos x
x 1 5. ln2 x – y ln x + x ln y = 1
3. y = (x + 1)2 (x – 2)3 Group 4:
Group 3: 1. y2 – 3xy + 4x – 5y + 7 = 0
2. x cot y – y sec x + x = 1
3
1. y = x3 4. y = (sin x – x)2 3. xe–2y – yey + 2x = 0
x 1 4. ey sec x – yex = 2
2. y = 5. y = xe–2x + sin2 x
x 1 5. y ln x2 – ln2 y = 1
3. y = (x + 1)3 (x – 2)2
Group 5:
Group 4: 1. x2 + y2 – 5x – 6y + 3 = 0

1. y = x3 x 4. y = (tan x – x)2 2. x sec y – y tan x + x = 1


3. e2x – x2ey + 2 = 0
1 x
2. y = 5. y = xe2x + sec2 x 4. exsin y – xey = 2
2 x
3 2
5. ln x2 – x ln2 y = 1
3. y = (x – 2) (x + 1)
nd
Test 5. Find the 2 derivative of the functions in
Group 5: Test 2.
Test 6. Find the six partial derivatives.
1. y = x x3 4. y = (ln x – x)2
Group 1:
1 x 2
2. y = 5. y = x ln x + cos x 1. z = y3 – 2x2y + 5x – 6y + 7
2 x
3. y = (x – 2)3 (x – 1)2 2. z = exy + e(x – 2y) + e3x – e2y – 5
3. z = sin (xy) – y sin x + sin x – cos y + 1
Test 3. Find the value of y’(1) in Test 2.’
Group 2:
Test 4. Find the derivative in differential form. 1. z = x3 + 3xy2 – 4xy + 5x + 7
Group 1: 2. z = e–xy + e(x + 3y) + e2y – 5

1. x2 – 3xy + 4y2 – 5x + 1 = 0 3. z = sin (xy) – x cos y + sin x cos y + 1

2. x sin y – y cos x + y = 1
57
Group 3:
1. z = y3 – 3xy2 – 2xy – 6y + 7
2. z = exy + e(x – 3y) + e3x – 5 Group 3:

 x 3  4x
3. z = sin x cos y + x cos y + sin x + 1 1
-1 ( x
2  3x  4) dx 2 x dx
1. 6.

Group 4: 2 x
0 (e  e  x ) 2 dx
x
3 1  9x 2
3 2 2. 7.
1. z = x – 2x y + 5x – 6y + 7
2. z = e–xy + e(2x + y) – e2y – 5
x e
1
0 (cos x  1)
2 dx 3 x
3. z = sin x cos y + y sin x – cos y + 1 2 3. 8. dx

1 ( x 2  1) dx
  4x 2  9
Group 5: dx
4. 9.
1. z = x2y + xy2 – 4xy + 5x + 7 0 x3  3 x  1

2. z = e(2x – 3y) + e3x – e2y – 5


  x
x
5. 1  tan x sec2 x dx 10. dx
3. z = cos x sin y + sin2 x – cos y + 1 x

Test 7. Evaluate the integrands. Group 4:

 x3  2 x 2
3
1 ( x
Group 1: 2  4 x  1) dx 2 x dx
1. 6.

 x3  3 x 2  2 x
2
-1 ( x
2  3x  2) dx 2 x dx
1. 6.
-2
0 (e x  e  x ) 2 dx
x
2
2. 7. 9 x 2  4 dx
x 2dx

2 x
2.  (e  e  x ) 2 dx 7. 
0 (cos x  1) x
2
0
4 x 2 1 3. 2 dx
8. ln 2 x dx

1
0 (sin x  1) x 2 (e2 x  1) dx x 2dx
2 dx 2
e 
3. 8. sin 2 x dx
4. 9.
0 2x  2x  3 9 x 2 1
2 ( x  1) dx
 x
2 9  4 x2
4. 9. dx
  x
0 x2  2 x  3 x
5. cot x  1 csc2 x dx 10. dx
x
 
x
5. sin x sin 2 x dx 10. dx
x 1 Group 5:

 x3  2 x 2
3
1 ( x
Group 2: 2  2 x  5) dx 2 x dx
1. 6.

 x 3  x 2  2x
3
-1 ( x
2  3x  4) dx 2 x dx
1. 6.
9 x2  4

1
2. -1 (e x  e  x ) 2 dx 7.
x2

-2
0 (e x  e  x ) 2 dx
x dx
2. 7.
x2  9 1
0 (tan x  1)  csc
2 dx 3
3. 8. 2 x dx
1 2 x
0 (sin x  1) e
2 dx sin x dx
3. 8. 1 (cos x  1) dx x 3 dx
2 (2 x  11) dx
4.
0 sin x  x  1
9.
 9x 2  4
 
dx
4. 9.
0 x2  x  2
4  9x 2 x
5.  1  sec x sec x tan x dx 10.  x x
dx

  x
x
5. cos x sin 2 x dx 10. dx
x

58
EVALUATION AND FEEDBACK

In order for your Sir Anacta to gain feedback on whether or not the objectives of this module were
attained (and for him to do some remedial measures if necessary), you are required to accomplish
the table below by putting a check mark (✓) on any one in the last five columns of each item. Don’t
worry, your negative answers will not be used against you. Just be honest and true to yourself.
Please send a screenshot copy (or the table itself) to him immediately after completing this module.
✂ ✂ ✂

After using this Module 1, I was Please check one for each item
No. Strongly Strongly
able to: Agree Neutral Disagree
Agree Disagree
Identify and distinguish the types of
1.
constants;
Familiarize and apply the derivative form
2.
of differentiation formulas;
Familiarize and apply the differential form
3.
of differentiation formulas;
Differentiate functions using implicit
4.
differentiation;
Differentiate functions using explicit
5.
differentiation;
Integrate functions using the Integration
6.
Formulas;
Integrate functions using the Integration
7.
methods and procedures;
About this module:
The topics were clearly and simply
8.
presented;
9. The contents are relevant and useful;
10. Some of the topics are new to me;
The pre-test, assessment tests and
11.
questions are relevant and useful;
Illustrations, figures and examples are
12.
clearly presented;
The bonus problems are challenging and
13.
motivating;
14. I benefited a lot from this module;
I will recommend this module to other
15.
students having this subject.

Any suggestion on how to improve this module? ______________________________


_____________________________________________________________________
_____________________________________________________________________

Evaluated by: ____________________ Course/Year: ____________


Name (Optional)

59
SUGGESTED READINGS*

th
1. James Stewart, Lothar Redlin and Saleem Watson. Algebra and Trigonometry. 4 ed., Cengage
Learning, 2016, pp. 183 – 621.
2. National Council for Teachers. Algebra 2 and Trigonometry. 2012, pp. 119 – 184.
rd
3. Beecher, Penna and Bittinger. Algebra and Trigonometry. 3 ed. Addison Wesley, 2007, All
pages.
th
4. Gustafson, Frisk and Hughes. College Algebra. 10 ed., Cengage Learning, 2010, pp. 260 – 450.
th
5. Aufmann, Barker and Nation. College Algebra and Trigonometry. 7 ed, Brooks/Cole, Cengage
Learning, 2011, pp. 153 – 559; 633 – 677.
6. Narasimhan. College Algebra and Trigonometry, Building Concepts and Connections.
Houghton Mifflin Harcourt Publishing Company. 2009, pp. 11 – 151.
th
7. Judith A. Beecher, Judith A. Penna and Marvin L. Bittenger. Algebra and Trigonometry. 4 ed.,
Addison-Wesley, 2012, pp. 1 – 648.
th
8. Barnett, Ziegler, Byleen and Sobecki. College Algebra with Trigonometry. 9 ed., McGrawHill
Companies, Inc., 2008, pp. 1 – 84; 109 – 460.
th
9. Van Dyke, Rogers and Adams. Fundamentals of Mathematics. 9 ed. Thompson, Brooks/Cole,
2007, All pages.
10. Research & Education Association. Algebra and Trigonometry - Problem Solver. 2004, All
pages.
th
11. Swokowski and Cole. Algebra and Trigonometry. 12 ed., Brooks/Cole,Cengage Learning,
2010, pp. 1 – 51, 123 – 211.
12. OpenStax College. Algebra and Trigonometry. Rice University, 2015, pp. 225 -774.
13. M. Adhikari and A. Adhikari. Basic Modern Algebra with Applications. Springer, India, 2014,
pp. 1 – 25.
14. I.M. Gelfand and M. Saul. Trigonometry. Birkhauser. 2001, All pages.
15. Stan Gibilisco. Trigonometry Demystified. McGraw Hill. 2003, pp. 1 – 121.
16. Louis Allen Hopkins and Earle Raymon Ziwet. Analytic Geometry and Principles of Algebra.
Wentworth Press, All pages.
nd
17. George F. Simmons. Calculus with Analytic Geometry. 2 ed. Mc Graw-Hill, 2001, pp. 1 - 46.
th
18. Howard Anton, Irl Bivens and Stephen Davis. Calculus with Analytic Geometry. 8 ed. John
Wiley and Sons, 2005, pp. 1 – 69.
rd
19. Barnett Rich and Philip A. Schmidt. Theory and Problems of Elementary Algebra. 3 ed.,
McGraw-Hill, 2004, pp. 315 – 330.
20. P. R. Vittal. Analytical Geometry 2D and 3D. Pearson. 2015, All pages.
nd
21. Titu Andreescu and Dorin Andrica. Complex Numbers from A to … Z. 2 ed., Springer, 2014,
pp. 1 – 22.

* e-copy of these books may be given to you upon request.

60
ATTACHMENTS

A. Symbols and Acronyms


 degrees; open interval HDE Homogeneous differential equations
 closed interval IC initial condition
 infinity IF integrating factor
< less than IFF if and only if
< less than IFFI Integrating factor found by inspection
> greater than indet indeterminate
 equal or less than to ITO in terms of
= equal to IV independent variable
 flow of solution/presentation k y-coordinate of a point
 approximately equal to ln Natural logrithm
a constant LDE Linear differential equations
3D three dimensional LHS left hand side
e Naperian logarithm base (2.7183) m constant
exp Exponential function n constant
a semi-major axis NM Numerical methods
ODE ordinary differential equation
 pi (3.1516)
OP ordered pair
2D two dimensional
P point
3D three dimensional
PDE partial differential equation
AAC Appropriate arbitrary constant
PS particular condition
BC boundary condition
r radius
BDE Bernoulli’s differential equation
Rad radian
c Arbitrary constant
RCS rectangular coordinate system
CL2V Coefficient linear in two variables
SSE Substitution suggested by the eq’n.
DE differential equation
SOV Separation of Variables
DV dependent variable
TA transverse axis
DIF Determination of integrating factor
WRT with respect to
EDE Exact differential equation
u dummy variable / integrating factor
e Naperian logarithm base (2.7183)
v dummy variable / integrating factor
f(x) function of x
GS general solution  pi (3.1516)
h x-coordinate of a point

B. Algebra, Trigonometry and Analytic Geometry

n/0 ln 0 log 0 ln (–a)


Special Values:
1
2
ln e = 1 log 10 = 1 tan(/2) i = –1 i=
–
0/n = 0 n/ = 0 a =0
Trigonometric Functions:
 =1
0 0
n =1 ln 1 = 0
Angle Trigonometric Function
/n =  n =   = 
Deg Rad Sin Cos Tan
Indeterminate Forms: 0 0 0 1 0
0/0 / 0* – 30 /6 0.5 0.866 0.577


0 0
0 1 45 /4 0.707 0.707 1
60 /3 0.866 0.5 1.732
Undefined/Imaginary Forms: 90 /2 1 0 

61
120 2/3 0.866 –0.5 –1.73 a/b a a ac
16. = 17. =
c bc b/c b
135 3/4 0.707 –0.707 –1
150 5/6 0.5 –0.866 –0.577 a/b ad ab a b
18. = 19. = 
180  0 –1 0 c /d bc c c c
210 7/6 –0.5 –0.866 0.577
a c ad  bc
225 –0.707 –0.707 1 20.  =
5/4 b d bd
240 4/3 –0.866 –0.5 1.732
270 3/2 –1 0  Binomial Exapansion:
2 2
300 5/3 –0.866 0.5 –1.732 21. a – b = (a + b)(a – b)
315 7/4 –0.707 0.707 –1 3 3 2 2
22. a + b = (a + b)(a – ab + b )
330 11/6 –0.5 0.866 –0.577
3 3 2 2
360 2 0 1 -0 23. a – b = (a – b)(a + ab + b )
4 4 2 2 2 2
Note: 24. a – b = (a + b )(a – b )

1 = 1 deg = /180 rad 1 rad = 180/ deg 2 2


25. (a + b) = a + 2ab + b
2

1 1 1 2 2 2
cot A = sec A = csc A = 26. (a – b) = a – 2ab + b
tan A cos A sin A
3 3 2 2 2
27. (a + b) = a + 3a b + 3ab + b
Algebra 3 3 2
28. (a – b) = a – 3a b + 3ab – b
2 2

Powers: 2
29. ax + bx + c = 0
–m m m m m
1. a = 1/a 2. (ab) = a b
2
m n m+n mn m n n m
c = b /4a (for prefect square trinomial)
3. a a = a 4. a = (a ) = (a )

a an
n
am 1  b  b 2  4ac
5.    6. =a
m–n
= x=
b bn an a nm 2a

   
m 1
m
7. a n = a m n = a1 n Page 52 of 54 Logarithms:
ln A n
30. e =A 31. ln A = n ln A
Radicals:
m
32. ln  A m  =   ln A
n
n
8. an = a 9. n ab = n a n b   n
mn m n
10. m a n b = a b 11. m n a = mn a 33. ln(AB) = ln A + ln B
p n
 A
12.  a m  = a mp
n n a a
13. n = 34. ln   = ln A – ln B
  b n
b B

mn
am am n
a Trigonometry
14. mn = =
n mn m
b bn b
Conversion:
1
1 m 35. sin  = tan  cos  =
15.
n
a m
=  a
n m 
=  a

m n
 =

an csc
1
36. cos  = cot  sin  =
Fractions: sec

62
1 sin  Analytic Geometry
37. tan  = =
cot  cos 
Straight Line:
1 cos 
38. cot  = = 62. Ax + By = C (General Form)
tan  sin 
y 2  y1
39. sec  = tan  csc  =
1 63. y – y1 = ( x – x1) (2-point Form)
cos  x 2  x1

1 64. y – y1 = m( x – x1) (Point-slope Form)


40. csc  = cot  sec  =
sin 
65. y = mx + b (Slope-y-intercept Form)
Identities:
66. y = m(x – a) (Slope-x-intercept Form)
41. sin  csc  = 1 42. sin  + cos  = 1
2 2

x y
43. cos  sec  = 1 44. sec  – tan  = 1
2 2
67. + =1 (Intercept Form)
a b
45. tan  cot  = 1 46. csc  – cot  = 1
2 2

68. x cos  + y sin  =  (Normal Form)


Multiple Angles:
47. sin 2 = 2 sin  cos  x y 1
69. x1 y1 1 =0 (Determinant Form)
48. sin 3 = 3 sin  – 4 sin 
3

x2 y2 1
49. sin 4 = 2 sin 2 (1 – 2 sin )
2

50. cos 2 = sin  – cos  = 1 – 2 sin  Circle: Center at (h, k)


2 2 2

= 2 cos  – 1
2 2 2
70. x + y + Dx + Ey + F = 0
51. cos 3 = 4 cos  – 3 cos 
3
2 2 2
71. (x – h) + (y – k) = r
2
52. cos 4 = 1 – 2 sin 2
Parabola: Vertex at (h, k) Opens
2 tan 
53. tan 2 =
72. Ax + Dx  Ey + F = 0
2
1  tan 2  Down/Up
73. By  Dx + Ey + F = 0
2
Left/Right
3 tan  tan3 
54. tan 3 = 74. (x – h) = 4a(y – k)
2
Up/Down
1  3 tan 2  Page 53 of 54
75. (y – k) = 4a(x – h)
2
4 tan  sec2  Right/Left
55. tan 4 =
tan 4   6 tan 2   1 Ellipse: Center at (h, k)
2 2
76. Ax + Cy + Dx + Ey + F = 0
Sum and Difference:

56. sin ( + ) = sin  cos  + cos  sin  ( x  h) 2 ( y  k)2


77. + =1
a2 b2
57. sin ( – ) = sin  cos  – cos  sin 
58. cos ( + ) = cos  cos  – sin  sin 
Hyperbola: Center at (h, k)
2 2
59. cos (– ) = cos  cos  + sin  sin  78. Ax – Cy + Dx + Ey + F = 0

tan  tan 
60. tan ( + ) = ( x  h) 2 ( y  k)2
1  tan tan  79. – =1
a2 b2
tan  tan 
61. tan ( – ) =
1  tan tan 

63
C. Differential Calculus

A. Derivatives – Direct Application / Single Variable

d d  c  2 c
c=0 (D-1)   = (D-7b)
dx dx  x2  x3
d d 
[c f(x)] = c  f ' ( x) = c f’(x) (D-2) d  c  3c
dx  dx    = (D-7c)
dx  x3  x4
d dx
x= =1 (D-3)
dx dx d  c  cn
 = (D-7d)
n
d
dx x  xn1
[f(x)  g(x)] = f’(x)  g’(x) (D-4)
dx d x
a = ax ln a (D-8)
dx
x = n  x n1
d n
n  0, 1 (D-5)
dx x
d a
a x = ln a (D-8a)
1 dx 2 x
d
x = (D-5a)
dx 2 x d x
e = ex (D-9)
dx
d c c
=  (D-5b)
dx x 2 x3 d e x
e x = (D-9a)
dx 2 x
d 3 1 1
x = = (D-5c)
dx 3x 2 / 3 3 2
3 x d 1
(loga x) = , x>0 (D-10)
dx (ln a) x
d c c
=  (D-5d) d 1
3
dx x 3 (ln x) = , x>0 (D-11)
3 x4 dx x
d n
dx
 
x =
1
n n1
(D-5e) d
dx
(ln |x|) =
1
x
, x 0 (D-11a)
n x
d 1
d c c (ln cx) = , (D-11b)
=  (D-5f) dx x
dx n x n n 1
n x d 2
(ln x2) = , (D-11c)
dx x
 x  p q  / q
d q p p
x = (D-5g)
dx q d n
(ln xn) = , (D-11d)
dx x
d c cp
=  (D-5h)
q
x pq
dx q 2 ln x
xp q d
(ln2 x) = , (D-11e)
dx x
d
 f (x)  g( x) =  f ( x)  g ' ( x) +  f ' ( x)  g ( x)
n ln n1 x
dx d
(lnn x) = , (D-11f)
dx x
(D-6)
d 1
ln x = (D-11g)
d  f ( x )   g ( x)  f ' ( x )  f ( x)  g ' ( x)  dx 2x
 =  
dx  g ( x )  
 g ( x)2 
d 1
(D-7) ln n x = (D-11h)
dx nx
d c c
 = (D-7a) d
dx  x  x2 (sin x) = cos x (D-12)
dx

64
d cos x d 1
sin x = (D-12a) (arcsec x) = (D-22)
dx dx x x2  1
2 x
d d 1
(cos x) = –sin x (D-13) (arccsc x) = – (D-23)
dx dx x x2  1
d
d sin x (sinh x) = cosh x (D-24)
cos x =  (D-13a) dx
dx 2 x
d
(cosh x) = sinh x (D-25)
d dx
(tan x) = sec2 x (D-14)
dx
d
2 (tanh x) = sech2 x (D-26)
d sec x dx
tan x = (D-14a)
dx 2 x d
(coth x) = –csch2 x (D-27)
dx
d
(cot x) = –csc2 x (D-15)
dx d
(sech x) = –sech x tanh x (D-28)
dx
d csc2 x
cot x =  (D-15a) d
dx 2 x (csch x) = –csch x coth x (D-29)
dx
d
(sec x) = sec x tan x (D-16) d 1
dx (arcsinh x) = (D-30)
dx
x 2 1
d sec x tan x
sec x = (D-16a)
dx 2 x d 1
(arccosh x) = – ,x>1 (D-31)
dx
d x 2 1
(csc x) = –csc x cot x (D-17)
dx
d 1
(arctanh x) = , |x| < 1 (D-32)
d csc x cot x dx 1 x 2
csc x =  (D-17a)
dx 2 x
d 1
(arccoth x) = , |x| > 1 (D-33)
d
(arcsin x) =
1
(D-18)
dx 1 x 2
dx 1  x2
d 1
(arcsech x) = ,0<x<1 (D-34)
d 1 dx
(arccos x) = – (D-19) x 1 x 2
dx 1  x2
d 1
d 1
(arccsch x) =  ,x0 (D-35)
dx
(arctan x) = (D-20) x 1 x 2
dx x2  1

d 1
(arccot x) = – (D-21)
2
dx x 1

B. Derivatives – Indirect Application / Function of Variables

Let: u = f(x) and v = f(x)


d du dv
(u  v) =  (𝒟-4)
d dx dx dx
c=0 (𝒟-1)
dx
du
u = n  u n 1
d n
du
n  0, 1 (𝒟-5)
d
(c u) = c  (𝒟-2) dx dx
dx dx
d 1 du
u = (𝒟-5a)
d du dx 2 u dx
u= =1 (𝒟-3)
du du
65
d c c du
=  (𝒟-5b) d du
dx u dx (tan u) = (sec2 u) (𝒟-14)
2 u3 dx dx

d 3 1 du d du
u = (𝒟-5c) (cot u) = –(sec2 u) (𝒟-15)
dx 3 2 dx dx dx
3 u
d du
d c c du (sec u) = (sec u tan u ) (𝒟-16)
=  (𝒟-5d) dx dx
3
dx u 3 4 dx
3 u
d du
(csc u) = –(csc u cot u) (𝒟-17)
d n
dx
 
u =
1 du
n n 1 dx
(𝒟-5e)
dx dx
n u d 1 du
(arcsin u) = (𝒟-18)
dx 1 u 2 dx
d c c du
=  (𝒟-5f)
dx n u n n 1 dx
n u d 1 du
(arccos u) = – (𝒟-19)
dx 1 u2
dx
 u  p q/ q
d q p du
up = (𝒟-5g)
dx q dx d du
1
(arctan u) = (𝒟-20)
2
d c du cp dx u  1 dx
=  (𝒟-5h)
dx q p q p  q dx
u q u d 1 du
(arccot x) = – (𝒟-21)
2
dx u  1 dx
d dv du
(u  v) = u  + v (𝒟-6)
dx dx dx d 1 du
(arcsec x) = (𝒟-22)
dx 2
u u 1
dx
du dv
v u
d u dx dx
 = (𝒟-7) d 1 du
dx  v  v2 (arccsc x) = – (𝒟-23)
dx x x 2  1 dx
d c  c du d du
 = (𝒟-7a) (sinh u) = (cosh u ) (𝒟-24)
dx  u  u 2 dx dx dx

d  c  2c du d du
  = (𝒟-7b) (cosh u) = (sinh u) (𝒟-25)
dx  x2  x3 dx dx dx

d du
d  c  3c du (tanh u) = (sech2 u) (𝒟-26)
 = (𝒟-7c) dx
 3  dx
dx u  u 4 dx
d du
 c  (coth u) = –(csch2 u) (𝒟-27)
 = cn
d du
 (𝒟-7d) dx dx
 n  n 1
dx u  u dx
d du
(sech u) = –(sech u tanh u ) (𝒟-28)
d u du dx dx
a = au (ln a) (𝒟-8)
dx dx
d du
(csch u) = –(csch u coth u) (𝒟-29)
d u du dx dx
e = eu (𝒟-9)
dx dx
d 1 du
(arcsinh u) = (𝒟-30)
d 1 du dx dx
(loga u) = (𝒟-10) u 12
dx (ln a)u dx
d 1 du
d 1 du (arccosh u) = – (𝒟-31)
(ln u) = (𝒟-11) dx dx
dx u dx u 12

d du d 1 du
(sin u) = (cos u) (𝒟-12) (arctanh u) = (𝒟-32)
dx dx dx 1 u 2 dx

d du
(cos u) = –(sin u) (𝒟-13)
dx dx

66
d 1 du
(arccoth u) = (𝒟-33) d 1 du
dx 1  u dx
2 (arccsch u) =  (𝒟-35)
dx dx
u 1 u 2

d 1
du
(arcsech u) = (𝒟-34)
dx dx
u 1 u 2

D. Differentials
Let: u = f(x) and v = f(x)
 c  cn
dc = 0 (D-1) d   = du (D-19a)
 un  u n1
d (c u) = c du (D-2a)
d u
d (u  v) = du  dv (D-4a) a = au ln a du (D-20a)
dx
d (un) = nun–1du , n  0, 1 (D-5a) u
d a
a u = ln a du (D-21a)
c dx 2 u
d= du (D-6a)
2 u
du
d (eu) = eu (D-22a)
c c dx
d =  du (D-7a)
u 2 u3 e u
de u = du (D-23a)
3 1 2 u
d u = du (D-8a)
3
3 u2 1
d (loga u) = du, u > 0 (D-24a)
(ln a)u
c c
d =  du (D-9a)
3 3 1
u 3 u4 d (ln ux) = du, u > 0 (D-25a)
u
 
d nu =
1
du (D-10a)
n u n1 1
n
d (ln cu) = du, (D-26a)
u
c  = 
c 2
d
n
du (D-11a) d (ln u2) = du, (D-27a)
u n1
u n u
n
n
 u  p  q  / q du
q p d (ln un) = du, (D-28a)
d up = (D-12a) u
q
2 ln u
c cp d (ln2 u) = du, (D-29a)
d =  du (D-13a) u
u pq
q q
up q
n ln n1 u
d (lnn u) = du, (D-30a)
d (uv) = u du + v du (D-14a) u
u v du  u dv 1
d  = (D-15a) d ln u = du (D-31a)
v v 2 2u

 c  c d ln n u =
1
d = du (D-16a) du (D-32a)
 u  x2 nu
d (sin u) = cos u du (D-33a)
 c  2 c
d   = du (D-17a)
 u2  u3 cos u
d sin u = du (D-34a)
2 u
 c  3c
d   = du (D-18a)
 u3  u 4 d (cos u) = –sin u du (D-35a)

67
sin u 1
d cos u =  du (D-36a) d (arcsec u) = du (D-49a)
2 u u u 12

2
d (tan u) = sec u du (D-37a) 1
d (arccsc u) = – du (D-50a)
sec 2
u u u2 1
d tan u = du (D-38a)
2 u d (sinh u) = cosh u du (D-51a)
d (cot u) = –csc2 u du (D-39a) d (cosh u) = sinh u du (D-52a)
2
csc2 u d (tanh u) = sech u du (D-53a)
d cot u =  du (D-40a)
2 u d (coth u) = –csch2 u du (D-54a)

du d (sech u) = –sech u tanh u du (D-55a)


d (sec u) = sec u tan u (D-41a)
dx d (csch u) = –csch u coth u du (D-56a)
sec u tan u 1
d sec u = du (D-42a) d (arcsinh u) = du (D-57a)
2 u u2 1
d (csc u) = –csc u cot u du (D-43a) 1
d (arccosh u) = – du (D-58a)
csc u cot u u 12
d csc u =  du (D-44a)
2 u 66
1
d (arctanh u) = du, |x| < 1 (D-59a)
d (arcsin u) =
1
du (D-45a) 1 u2
1 u2 1
d (arccoth u) = du, |x| > 1 (D-60a)
1 1 u2
d (arccos u) = – du (D-46a)
1 u2 1
d (arcsech u) = du (D-61a)
1 u 1 u 2
d (arctan u) = du (D-47a)
u 1
2
1
d (arccsch u) =  du (D-64a)
1 u 1 u2
d (arccot u) = – du (D-48a)
u2 1

D. Integral Calculus

Note: Constant of integration c is omitted in each formula.

= ln  u  u 2  a 2 

du
I-0: c =0 I-30:
u a
2 2  


u du
I-1:  du =u I-31: = u2  a2
u a
2 2

I-2:  c du = cu I-32:  sin u du = –cos u

au
 a du I-33:  cos u du = sin u
u
I-3: =
ln a

u n 1
 u du  tanu du
n
I-4: = I-34: = –ln |cos u|
n 1
2
I-5:  u du =
3
u u I-35:  cot u du = ln |sin u|

68
e I-36:  secu du = ln |sec u + tan u
u
I-6: du = eu

e nu
I-7:  u e nu du =
n2
( nu  1) I-37:  cscu du = ln |csc u – cot u|

u
du 1 1
 sin
I-8: = ln u I-38: 2 u – sin 2u
u du =
2 4

 ln u du = u ln u – u
1

3 3
I-9: I-39: sin u du = cos u – cos u
3
u2 1 1
I-10:
 u ln u du =
4
(2 ln u – 1) I-40:  cos
2
u du =
2
u + sin 2u
4
ln 2 u

ln u 1
 cos u du =  sin 3 u + sin u
I-11: du = I-41: 3
u 2 3

 u ln u = ln (ln u)
du
 tan
2
I-12: I-42: u du = tan u – u

 u = ln u
du 1
 tan tan 2 u + ln (cos u)
I-13: I-43: 3
u du =
2


du
 cot
2
I-14: =2 u I-44: u du = –cot u – u
u

u
du 1 1
 cot cot 2 u – ln (sin u)
3
I-15: =– I-45: u du = 
2 u 2

( au  b) n 1
I-16:
 ( au  b ) n du =
a ( n  1)
I-46:  sec
2
u du = tan u

 au  b
du 1
 csc
2
I-17: = ln |ax + b| I-47: u du = –cot u
a

 au  b du = a – a ln |ax + b|
u u b 1 1 1
I-18: I-48:  sin u cos u du = sin2 u = – cos2 u = cos 2u
2 2 2 4

 (au  b) du = a (au  b) + a
u b 1
I-19: 2 2
ln |ax + b| I-49:  sin u tan u du = ln |sin u + tan u| – sin u
2

au  b
 u(au  b)
du 1
I-20: =– ln I-50:  sin u cot u du = sin u
b u

au  b
u
du 1 a
I-21: =– + ln I-51:  sin u csc u du = u
2
(au  b) bu b 2 u

au  b
u
du 2a
I-22: = ln I-52:  sin u sec u du = –ln |cos u|
2
(au  b) 2
b 3 u
a 1
– – I-53:  cos u tan u du = –cos u
b (bu  a)
2
b 2u


u 2
I-23: a 2  u 2 du = a  u2 I-54:  cos u sec u du = u
2
a2 u
+ arcsin I-55:  cos u cot u du = –ln |csc u + cot u| + cos u
2 a

a
du 1 u
I-24:
2
u2
=
a
arctan
a
I-56:  tanu sec u du = sec u

69
1 
 u 2  a 2 du = u u  a I-57:  cos u csc u du = ln |sin u|
2 2
I-25:
2
 
 a2 ln  u  u 2  a 2 
 
I-58:  tanu sec u du = sec u

du 1 au
I-26:  a2  u 2 =
2a
ln
a u
I-59:  cot u csc u du = – csc u
ua
u
du 1
I-27: = ln I-60:  arcsin u du = u arcsin u + 1 u2
2
a 2 2a ua


du u
I-28: = arcsin I-61:  arccos u du = u arccos u – 1 u2
a u2 2 a

u
du
I-29: = arcsec u I-62:  arctanu du = u arctan u – ½ ln|u2 + 1|
a2  u 2

IMP-1: Integration by Parts (IbyP): N ( x) A B C


= + +…+
(ax  b) n ax  b (ax  b) 2
(ax  b) n
 u dv = uv –  u dv

Case 3: D(x) contains distinct quadratic factors:


IMP-2: Integration of Rational Fractions by Partial
Fractions (IRF by PF) N ( x)
N(x)/D(x)  PFs (ax2  bx  c)(dx2  ex  f )...(gx2  hx  i)
Ax  B Cx  D Ex  F
Case 1: D(x) contains distinct linear factors: = + +…+
ax 2  bx  c dx  ex  f
2
gx2  hx  i
N ( x) A B
= +
(ax  b)(cx  d )...(ex  f ) ax  b cx  d Case 4: D(x) contains repeated quadratic factors:

+…+
C N ( x) Ax  B Cx  D
= + +…
ex  f (ax2  bx  c)n ax 2  bx  c (ax2  bx  c)2

Case 2: D(x) contains repeated linear factors: Case 5: D(x) contains combination of factors.

IMP-3: Trigonometric Substitution

Integrand
Case Use subst. Apply
contains

1 a2 u2 x = a sin  a 2  u 2 = a cos 

2 u2  a2 x = a tan  u 2  a 2 = a sec 

3 u2  a2 x = a sec  u 2  a 2 = a tan 

IMP-6: Powers of Cotangents and Cosecants


IMP-4: Powers of Sines and Cosines (PSC) (PCC)
> Transform integrand into (I-4) and (I-31) to (I-54), > Transform integrand into (I-4) and (I-31) to (I-54),
whichever is applicable. whichever is applicable.

IMP-5: Powers of Tangents and Secants (PTS) IMP-7: Algebraic Substitution (AS)
> Transform integrand into (I-4) and (I-31) to (I-54), > Replace any quantity from the integrand by a
whichever is applicable. single variable.

70
About your Sir Ernesto Anacta

EDUCATION:
> Bachelor of Science in Civil Engineering (BSCE) Feati University
> Master of Science in Civil Engineering (MSCE – Structural) UP - Diliman
> Doctor of Philosophy in Civil Engineering (Ph.D.CE Structural) UP - Diliman

WORK EXPERIENCE:

ACADEMIC EXPERIENCE:
> Academic Rank: Professor 2
> Designation: Dean, College of Engineering
> Teaching Experience: 33 years

ADMINISTRATIVE EXPERIENCE:
> Vice President for Administration
> Assistant to the President for Administration
and Special Projects
> Chairperson, Bids and Awards Committee (BAC)
> Campus Administrator – ESSU Maydolong Campus
> Director, Research and Development
> Director, Procurement Services
> Director, Infra Development and Engineering Services
> Director, Physical Plant and Maintenance Services
> Chairman, Selection and Promotions Board
> Chairman, Grievance Committee

SIGNIFICANT ACHIEVEMENTS:
> First graduate of Ph.D. Civil Engineering of UP after 100 years of its existence
> First Ph.D. graduate of the Institute if Civil Engineering, UP-Diliman
> First graduate of Ph.D. Civil Engineering from a Philippine school
> First in ESSU - COE to present a paper in international convention

SCHOLARSHIPS:
> National State Scholarship Program
> Feati University Academic Scholar (consistent)
> ESSC Faculty/Staff Development Program
> DOST/ PCIERD Scholarship Program (MS and Ph.D. Program)
> UP-Diliman Graduate Studies – University Scholar
Education is the key to success in life,
and teachers make a lasting impact
in the lives of their students.
- Solomon Ortiz

WARNING: No part of this module shall be copied or reproduced in any manner


in whole or in part without written permission from the author.

You might also like